Practice Test / Test 1 / Diagnostic Explained - Kaplan Test Prep

17 downloads 879 Views 541KB Size Report
or incorporated into any information retrieval system, electronic or mechanical, without the written permission of Kaplan, Inc. LSAT is a registered trademark of ...
Practice Test / Test 1 / Diagnostic Explained December 2008 Exam plus experimental from June 2000 Exam

© 2012 Kaplan, Inc. All rights reserved. No part of this book may be reproduced in any form, by photostat, microfilm, xerography, or any other means, or incorporated into any information retrieval system, electronic or mechanical, without the written permission of Kaplan, Inc. LSAT is a registered trademark of the Law School Admission Council.

Section I: Logical Reasoning

Section I: Logical Reasoning 1. (E) Flaw This next LR section begins with a Logical Flaw question, so it pays to keep our eyes and ears peeled for the Classic flaws that appear perennially on the test. And we’re not disappointed: According to the argument, many people hear about research into ESP, and are thus convinced that ESP is real. The author, however, concludes that ESP is a myth, and that those people have been fooled, all based on evidence that one renowned researcher cheated on his or her experiments. In other words, the author attempts to debunk an entire field with evidence concerning only one specific case. In LSAT lingo, we call this a “non-representative sample,” and it’s a faulty form of reasoning that’s not uncommon on the test. One simply can’t hold forth on the general state of ESP research on the basis of one specific example. “Overgeneralizes” is a perfect way to describe this flaw, so (E) gets the point. (A) The journalist doesn’t actually attack the integrity of the prominent researcher; the researcher himself admits to falsifying data. The problem lies in how the journalist interprets that fact. (B) bollixes up the elements of the argument. The journalist believes ESP is a myth because of the falsified data, not because the pubic believes in it. The notion of public belief comes up in a different context: The public believes in ESP because of the experiments they’ve heard about. (C) is tempting, since experiments are the only evidence for ESP that the author discusses. But the fact that the author links experiments and beliefs in this particular case is not to say that the author assumes in general that experiments provide the only possible evidence for beliefs.

increases calcium elimination in the intestines, and causes less calcium to go to the kidneys, then it’s much easier to understand how increasing calcium intake can actually decrease the chances that new stones will form. (A) First of all, animals are animals, and people are people, so kidney stones in animals are technically outside the scope. But, ignoring that, and assuming we can learn something about human kidney stones from animal studies, (A) merely repeats the oddity without going one step toward explaining it. (B) The passage says nothing about taking oxalate to help prevent the recurrence of stones. Also, we are interested in explaining how increasing, not reducing calcium intake reduces the chance that stones will form. (C) Even if this were true, and in some cases the disorder and the extra calcium are both caused by a third factor, the fact still remains that stones can form, as the first sentence states, when urine is overly concentrated with calcium. We’d still be in a position to wonder why increasing calcium intake reduces the chances that new stones will form. (E) The scope of the passage is kidney stones formed by calcium and oxalate, not by uric acid. This explains nothing about how increasing calcium intake helps prevent stones from reforming. 3. (C) Inference Next up we’re looking for something that can be inferred from a conversation. David thinks that not allowing companies to hire permanent replacements for striking employees is unfair, since that would place them in a weak bargaining position with the strikers. Lin claims that companies would maintain leverage if they hire temporary replacements. It’s not easy to prephrase the correct inference, so let’s evaluate the choices, keeping a close eye on the scope of each statement.

(D) is way too extreme: Only the specific scientist in the argument is presented as unreliable. The author implies nothing about scientists in general.

(A) The scope of David’s argument is limited to the option of hiring permanent replacements. It’s impossible to know what effect he thinks temporary replacements will have on the situation.

2. (D) Paradox

(B), (D) The speakers argue over what will be the case in the event of a strike, not what should be the case, so both of these fall outside the scope. True, David finds the situation “unfair,” but there’s still no way to know from these statements exactly how much leverage these two think companies should have. So we can’t say that they believe the leverage should be equal (B), nor can we infer that they disagree on the relative amount of leverage each group should have (D).

“Resolve the apparent discrepancy” in the stem signals a Paradox question, so we need to locate the mystery, paraphrase it in user-friendly terms, and think about possible solutions. Why, if excess calcium in the urine forms kidney stones, is the chance of recurrence decreased by adding more calcium to one’s diet? It may seem that more calcium in the diet would mean more in the urine, but nowhere does it say that the amount of calcium in the urine is directly proportional to the amount of calcium one consumes. Perhaps there’s a mechanism that causes an increase in dietary calcium to lead to a decrease in urinary calcium. Perhaps increasing calcium intake has some other, unmentioned effect, which reduces the chances that calcium in the urine will form stones. (D) provides such an alternative: If increasing calcium intake

(C) fits the bill: While David believes companies will have “little” leverage if they can’t hire permanent replacements, Lin thinks hiring temporary replacements would afford them “sufficient” leverage in their battle with strikers. So David and Lin disagree over how much leverage companies will lose in a strike situation if forbidden to hire permanent replacements.

3

LSAT Practice Test Explained

(E) describes David’s position. If anything, it would be easier to infer the opposite of (E), considering that Lin believes there’s an alternative to hiring permanent employees that wouldn’t entail sacrificing bargaining power. 4. (E) Principle (Strengthen the Argument) Here we need to locate a principle that would help justify the conclusion of the argument, so we’re essentially looking for a strengthener stated in general terms. The final sentence provides the conclusion: A biographer’s ignorance of wellknown facts can’t be used to escape blame for misleading readers. The second sentence is the key: “The book’s author claims that, because he was unaware of these facts when he wrote the book, he is not accountable for the fact that readers were misled by this omission.” The author feels that exactly the opposite is true; something like “A biographer is accountable for misleading readers if he is unaware of wellknown facts.” This is what choice (E) expresses. (A) and (C) can be safely omitted after reading “An author of a biography should not be blamed…” The gist here is that the author should be blamed. Furthermore, as for (A), the book’s popularity is clearly outside the scope. (B) weakens the author’s position that ignorance is a valid cause for blame. (D) is just plain wrong. Far from being the only one in possession of the incriminating facts in questions, the biographer pleads ignorance of them, so this principle doesn’t match the thrust of the argument. 5. (D) Flaw When we see “the logician’s argument is questionable because” in the question stem, we think “find the logical flaw.” And we have to admit the testmakers are not without a sense of irony, beginning a Logical Flaw question with the phrase “Having mastered the laws of logic. . .” Hmm…evidently not. So, because he has supposedly mastered logic, the logician claims that for him to commit a logical flaw in conversation would be like a physicist violating the everyday laws of physics. This seems fairly silly on its face. Does the logician mean that because no physicist can simply float off into space, for example, he, a master logician, cannot make a logical mistake? We can see immediately that the two situations are not the same. Violating the laws of physics is by definition impossible, whereas the logician can easily violate the laws of logic by misspeaking. The laws of physics and the laws of logic differ in important respects. The laws of physics apply universally and cannot be violated; the so-called “laws” of logic are learned and therefore can be violated. The two situations are not analogous, as choice (D) correctly points out. (A) and (B) raise irrelevant issues. How we perceive physical laws or how difficult physics is to master are unimportant—the relevant issue is the comparison between violating physical laws and violating the laws of logic.

4

(C)’s a tough choice, but strange as it may seem, the logician need not rule out this possibility in making his argument. It would be preposterous, a rare one-in-a-million occurrence were someone able to circumvent the laws of physics, but the logician is simply arguing that the odds are just as likely for that to happen as they are for him to misspeak. In making this comparison, he need not establish that it’s absolutely impossible for a physicist to cheat physical laws in order for this overall point to come through. (E) Again, tricky: Nothing explicitly stated in the conclusion contradicts the notion that our logician is a master of logic. Ah, but what about fact that the conclusion itself is flawed? Well, one can commit a logical error and still be (or at least consider oneself) an expert in logic. Arguments by analogy are common on the LSAT, and the first thing you should do when you see one is check to make sure that the things being compared are in fact comparable. You should be especially wary when an author argues by analogy in a Flaw question; it’s almost certain that the analogy won’t hold. In this case, a logician making a logical error in conversation is not analogous to a physicist violating the laws of physics. The first is a mistake; the second is an impossibility. 6. (D) Main Point We need to infer a conclusion from a survey, so let’s piece things together as best we can and then rigorously evaluate the choices. Just a bunch of facts: The sample was 1000 adults in Denmark, representative of the general population, and who happened to also be questioned ten years ago on the same issue: the banning of cigarette advertising. The results were little changed from the first survey: 31 percent in favor, 24 percent against, 38 percent in favor of a ban limited to certain media, and 7 percent no opinion. We can infer that roughly these same figures must have resulted ten years ago, since opinions have “changed little,” but there’s nothing really earth-shattering to prephrase from all this, so let’s hit the choices. (A) We discussed overgeneralization back in Q. 1 of this section, and here’s another perfect example. (A) would have us reason from a single survey result to a sweeping conclusion about the nature of people’s opinions in general. (B) The numbers merely reflect the percentage of those in favor and not in favor of the ban. The number against the ban constitutes a minority, sure, but how do we know they feel the ban would “set a bad precedent”? The survey doesn’t delve into the reasons for people’s opinions, so there’s no way we can jump to the conclusion here. (C) It’s hard to infer (C), given the fact that 69 percent of those surveyed want to ban the advertising in some form. Which brings us to. . . (D) has it right: As just noted, 69 percent of those surveyed want to ban cigarette advertising in at least some form, and

Section I: Logical Reasoning

69 percent qualifies as “most of Denmark’s population.” (See first bullet point below.)

change; she merely says that frequent changes are a moneymaking ploy.

(E) We don’t have any data on the number of people who smoke in Denmark, so this conclusion is unwarranted. The focus is on what people think about cigarette advertising; how many actually smoke is anyone’s guess.

(C), (E) Other purchasers besides libraries and libraries’ budgets for other books are both issues that fall outside the scope of the argument. The issue is the motive publishers have in selling updated systems to libraries, not to choice (C)’s other buyers. As for (E), the argument concerns only these classification books, so the ability to buy other books is irrelevant.

7. (B) Paradox This question is basically a Paradox question in disguise. It’s not obvious from the stem, but the author concludes that it is surprising that airports are expanding passenger terminals now, instead of five years from now when the number of passengers is expected to rise. The paradox arises because the author states that airport expansion is called for only when air traffic increases. But what if there is a valid reason to expand airport terminals when the number of passengers is low? That’s the kind of alternative possibility we’re looking for, and that choice (B) provides: It’s better to expand terminals when there are fewer people, because it will be less of an inconvenience to travelers. That’s certainly a relevant consideration; the industry wouldn’t want to aggravate people just as they’re flocking back to airports. Beginning the construction now can be justified by the projection of increased passengers down the road coupled with the practical consideration presented in (B). (A), (C) The economy in general is clearly outside the scope of the argument, while rising fuel costs may have some relevance, but we can’t tell for sure what effect it will have on the situation. If anything, this factor may throw the projections of future volume increases into question, which would only make it more surprising that airports are going ahead with the construction now. (D) Routes and planes? This mystery is about terminals. No help. (E) So, some airlines may go belly up. So what? This still doesn’t tell us why airports are planning to expand now, despite the fact that volume is not expected to return to previous levels for five years. 8. (B) Flaw In this flaw question, the author errs in arguing that the only possible reason to change the library classification systems is for publishers to make money. Perhaps there are other, less suspicious reasons for doing so. This simple notion is reflected in answer choice (B)—maybe libraries need the books for valid reasons. (A) and (D) both that claim the author does something that she has not done. The author never states that the new systems are unreasonably expensive, just very expensive. What’s unreasonable to her are the frequent changes, but that’s a different issue. And (D)’s use of the word “ever” is too extreme: The author never says that the systems must never

9. (A) Strengthen the Argument Isn’t there a song “California Dreaming”? We don’t think this is what they had in mind. We need to strengthen the argument that experiencing an earthquake can cause people to dream about earthquakes. The evidence is that almost none of the students in Ontario, who never experienced an earthquake, dreamt about earthquakes, while half the Californian students did dream about earthquakes after going through a major quake. Well, we know that we can weaken a causal argument by severing the link between the supposed cause and the effect, and one way to do that here is to show that students from California dreamed about earthquakes before the big one hit. Then it wouldn’t seem as likely that experiencing the quake caused the dreams. (Maybe something else, like media coverage from previous quakes, keeps Californians—even those who haven’t personally experienced one—dreaming of quakes all the time…) Since this would damage the argument, the author assumes that they didn’t dream much about quakes before the one in question, and the argument would be strengthened if this assumption were bolstered. Choice (A) accomplishes this. The Ontario group is the control group, so if the Californian dreams prior to the quake were similar to the Ontario dreams (almost none about earthquakes), then the possible alternative discussed above is discounted, the author’s assumption is shored up, and we’d be more likely to believe the conclusion in the final sentence. (B) It’s unclear what studying dreams and dream recollection has to do with actually recalling dreams, so it would take a lot of maneuvering to find a way for this fact to help the argument. Does taking such classes make people dream more? dream less? better able to remember dreams? There’s no way to know this, so we have to take a pass on this one. (C) Any effect these previous earthquakes had on the students or their dreams goes unmentioned. Previously experiencing earthquakes may certainly cause the Californian students to have earthquakes more firmly in mind than the Ontario students, but this can’t help the argument if we don’t know what effect these quakes had on their dreams prior to or during the study. (D) The number of overall dreams is irrelevant; it’s the number of students having dreams about earthquakes that matters. (E) is outside the scope, as the conclusion focuses on the relationship between dreaming about earthquakes and

5

LSAT Practice Test Explained

experiencing them. (We know California earthquakes can be big, but it’s doubtful the folks over in Ontario felt it.)

require intelligence. More importantly, (A) directly contradicts the evidence that states that a computer can never have emotions.

10. (D) Inference

(B) The conclusion that computers will never display intelligence is not based on the fact that today’s technology simply can’t hack it; if that were the argument, then maybe (B) would be a contender. The conclusion is based on specific evidence regarding emotions, so this stuff about technological advancements has no place in it. (Not convinced? Try the Denial Test: What if computer technology will soon advance way beyond the current state? Would that sink the argument? Would we now believe that maybe computers will be able to display intelligence? No; there’s still the little problem of their lack of emotions. We can’t ignore the evidence altogether and simply assume a little extra tech power will change everything. (B) fails the Denial Test because this is an argument about intelligence and emotions, not technology.)

Another Inference question, so our job is to interpret and combine statements. A few facts, pretty basic stuff: Wasting natural resources is wrong; burning lots of trash in incinerators wastes resources. We can combine these to infer that burning lots of trash in incinerators is wrong, but unfortunately that’s not among the choices. However, the correct inference is nothing more than a simple interpretation of the latter statement: If “it is an incredible waste of resources to burn huge amounts of trash in incinerators,” then it makes perfect sense that if it wishes to avoid wasting resources, the city can’t incinerate huge amounts of trash. (D) gets the point. (A) assumes that burning trash and recycling are the only two ways to dispose of trash. Maybe there are other methods of disposal such as landfills. (B), (E) Let’s assume for the sake of (E) that “not burning” means “recycling,” even though we just saw above in (A) that that need not be the case. Recycling causes fewer resources to be wasted, but that doesn’t necessarily mean there would be no waste at all. We don’t know that the city would be able to stop wasting resources entirely. Both of these choices are too extreme. (C) The author compares the results of recycling only to the results of incineration, and doesn’t mention any other methods of conservation. So we can’t infer that recycling is the most effective way to conserve resources. 11. (E) Assumption Did you pick up on the scope shift in this Assumption question? Did you notice the mismatched terms? If not, take another look before reading on. Hint: look closely. Okay: Human intelligence requires human emotions. A computer can’t have human emotions. Therefore . . . a computer can’t display human intelligence. What’s wrong with that? Nothing—but that’s not what the author says. She concludes that computers can’t display intelligence period; she leaves out the word “human,” and therein lies the shift. What if there are other kinds of intelligence that don’t require emotions? Then the author couldn’t argue that computers can never display intelligence based solely on the fact that they don’t have emotions. If the author wants to make a case about intelligence in general based on the evidence given, then she needs the connection cited in (E). (Note: If you missed the distinction between human intelligence and intelligence in general, (E) probably looked to you like a restatement of the first sentence.) (A) gets things backwards: The underlying mechanism here, whether you caught the subtle shift described above or not, is that intelligence requires emotions. (A) says that emotions

6

(C) and (D) are outside the scope: Identifying emotions (C) isn’t relevant to the argument; possessing them is. As for (D), we know human intelligence requires the capacity for emotions, but it’s not necessary for the amount of emotions and intelligence to increase proportionally. 12. (C) Weaken the Argument This Weaken question provides a virtual clinic in analyzing a study; four of the choices will weaken the conclusion based on the study results, while the right answer will be the choice that’s irrelevant, outside the scope, or even strengthens the argument. The conclusion is that most people would want to be told if they had a serious medical condition, based on the fact that 80 percent of those surveyed said that they would want to be told. When looking to weaken any argument that relies on a study, experiment, or survey, we look for choices that would invalidate (or at least seriously call into question) the results of the survey, and the weakeners here, as discussed below, fall into fairly common categories. The choice that does not weaken the argument predictably strays from the scope, and that’s choice (C): The background of the researchers has no impact upon how those surveyed chose their answers, or on how the author interprets the results. (A) Another survey yields the exact opposite conclusion. That can’t possibly bode well for the argument, especially since we aren’t given a way to assess the validity of either survey. It seems we can’t trust either one. (B) If there’s a good chance people didn’t reply honestly on the survey, as (B) suggests, this would clearly weaken the argument since the conclusion relies exclusively upon the survey results. (D) suggests the possibility that people were led by the survey itself to choose the notification option, since the questions themselves made this sound like the reasonable answer. (D) introduce a bias into the survey, which casts doubt on the

Section I: Logical Reasoning

validity of the survey and again would weaken the argument based on the survey results. (E) The conclusion asserts what “most people” would want. This choice tells us that the survey was limited to a specific type of person, not a characteristic cross-section of society. We should have known that we weren’t going to get out of this one without the appearance of the “non-representative sample.” 13. (C) Assumption Another standard Assumption question, and you know the drill: Paraphrase the components of the argument, then look for scope shifts, mismatched terms, and elements that need to be connected for the argument to work. The author states that totalitarian regimes that censor expression on moral grounds tend to expand the scope of the censored material to criticisms they perceive may threaten their power. The conclusion drawn from this, signaled by “Accordingly,” is that writings that could reduce passivity are vilified (labeled as blasphemous or pornographic) by many totalitarian regimes. If totalitarian regimes wish to censor, on moral grounds, expression that challenges their power, and therefore, according to the historian, vilify as immoral writings that make people less passive, then the historian must be assuming a connection between reduced passivity and a threat to the regimes’ power. This connection is forged in (C). (A) No kidding! But what constitutes popularity doesn’t help integrate the notion of reduced passivity into the framework of the argument. (B) Regimes other than totalitarian ones are outside the scope. Still, try the Denial Test if you like: What if all regimes that censor in this way are totalitarian? Does that sink the argument? No. The author never states that there are other kinds of regimes that do this. It’s not part of the argument, which brings us back to our initial critique: outside the scope. (D) is tempting! But the connection (D) forges between public passivity and power is too strong. The argument is concerned with perceived threats to power; the answer choice talks about what is actually needed to retain power. Also, “a regime” is a bit too broad here; it would be better to specify the kind of regime the historian is interested in, the totalitarian regime. (E) asserts that most material labeled immoral would in fact be effective in reducing passivity, but that only indicates that their assessments are on the money. But even if these writings wouldn’t have much effect, the regimes think they will, and we’re still left with the gap between their ideas on passivity and their perceptions of power. 14. (C) Main Point Next we’re asked to figure out the conclusion of the ethicist’s argument, so let’s paraphrase: Of two clot-dissolving agents, the costlier one, TPA, would save at most two more lives out of every 50 cardiac patients than would ASA when given after

an operation. But, since relatives of those who don’t survive surgery would feel worse if the less effective, cheaper ASA were given, one shouldn’t base the decision of which drug to use on financial concerns alone. Choices (B), (D) and (E) repeat the facts of the matter; they are the evidence for the conclusion summarized in (C). We know the last part of the last sentence is the conclusion because it is prescriptive; it says what the author thinks should be done, based on all of the evidence given before. It’s the only choice that passes the Kaplan One Sentence Test; that is, if all the author were allowed was one sentence, (C) is the statement that would best convey his point. (B), (D), and (E) are all too narrow to encapsulate the entire argument; as we saw, each is a piece leading up to the main point. (A) is an extreme choice that goes too far—there may be special issues with particular patients that require the use of ASA over TPA. Besides, the recommendation is simply to weigh the considerations, not to banish ASA altogether. 15. (A) Inference Here’s a question type that used to be more common on the test, the old “crossed-wires” example. One person speaks, the second responds based on a misunderstanding of what was said, and we’re asked to figure out what he was thinking in order to respond the way he did. So, how does Joshua interpret Ashley’s statement? As far as scope shifts go, this one is pretty blatant: Ashley says that the words “of” and “upon” don’t refer to anything. Joshua agrees, and says that because such words are meaningless, they should be abandoned. The logic of Joshua’s response clearly relies on the notion that if a word does not refer to anything, then it’s meaningless. That’s his interpretation of Ashley’s comment (although it’s not what she actually said). That could be the answer, but evidently the testmakers found that to be too easy, so instead they went with the contrapositive: “If a word has meaning, then it refers to something.” (A) is the equivalent of this in “only” form. (B), (D) and (E) refer to the “usefulness” of a word, but technically, usefulness is outside the scope. Ashley’s concerned with the kinds of words that do and do not refer to things, and Joshua picks up on this (albeit wrongly) and adds to it his notion of meaning. So you can eliminate these quickly based on scope alone. However . . . We could get picky and say, “hey—he’s saying that meaningless words should be abandoned, which is pretty much saying that they’re useless, right?” Let’s give that train of thought the benefit of the doubt, and infer that Joshua believes that meaningless words are not useful. There are two problems with this. First, while we may be able to infer this from his comment, that still doesn’t address his misunderstanding of Ashley’s “not refer to anything” argument. Secondly, this is not among the choices; not one of

7

LSAT Practice Test Explained

the three choices involving “usefulness” reflects this possible inference. All in all, they have to go. (C) looks very close to correct choice (A), with one major difference: the word “only.” Without this word, Joshua’s claim about meaninglessness has no context. If he merely heard her imply that “words that refer to something are meaningful,” he would have no basis for responding that words that don’t refer to anything are meaningless. Hearing in his mind that “only” is necessary for his response, which is why (A) is correct and (C) is not. 16. (D) Role of Statement Another ethicist? Has anyone ever actually met an ethicist? There seems to be a bunch of them running around somewhere. Anyhow… This time we’re asked to determine the role of a statement in the argument, so it’s best to keep our eye out for the argument’s structural components. Some would ban cloning because clones would be a lower class of people designed to indulge the vanity of the original person. The next sentence is the assertion we’re to categorize. It contains the crucial Keyword “however,” which essentially tells us that it will likely contain some kind of refutation of the first claim. And so it does: The counter is that we already use other people as a vehicle for personal ambitions, and it’s by no means illegal to do so. The ethicist provides an example of people who aren’t clones who are used to indulge other’s egos, but for our purposes we’ve already come upon the relevant issue: As the Keyword “however” suggests, the phrase in question supports the idea that using clones for vain purposes is not a sufficient reason to ban cloning. This is choice (D). (A) is way off base. The most concrete thing we can say about individuality (a concept not explicitly discussed and relevant here at best by implication) is that the opponents of cloning seem to value it. But what that has to do with the ethicist’s legal example is anyone’s guess. (B) First of all, no such conclusion exists in the argument; the children example is merely brought up to illustrate the point that some use others for their own vanity. Whether this is “objectionable” or not isn’t discussed. Moreover, the phrase concerning illegality is far removed from this example—the argument’s structure makes it clear that the assertion in question is there to respond to a previous point. (C) The ethicist doesn’t argue that clones are not “subpeople” but rather that “subpeople”—at least in the sense of those who exist as an extension of another’s ego—are not uncommon. (E) represents a classic distortion. The author cites that the practice in question is in fact common and legal, and uses this information as part of a challenge to the opponent’s position. The ethicist is far from arguing that using people as vehicles for personal ambitions should be made illegal.

8

17. (E) Assumption The stem tells us that Trent is the featured player in this one, but first Selena’s argument. In a nutshell, Selena thinks asteroid impact unleashed a chain of events that caused the dinosaurs to die out. Trent responds that the extinction of the dinosaurs must have been due to some cause other than asteroid impact, based on the facts that the crater Selena mentions is too small to create the amount of dust required in her scenario, and that the extinction took many years. Both of Trent’s facts seem to argue against the likelihood that the Mexican asteroid, by itself, did the dinosaurs in, but what if many asteroid impacts over time raised enough dust to set the process described in motion? (Notice that Selena never says that a single asteroid wiped out the dinosaurs.) If Trent accepted the possibility of multiple impacts, he’d have to rework his argument. For his argument as stated to stand, he’d have to assume that this didn’t happen—choice (E). (A) Land vs. ocean impacts? What does this distinction have to do with anything? Trent is trying to show how details regarding this particular Mexican crater and a long extinction time lead to some other cause for the extinction. Where asteroids hit in general has nothing to do with this. (B) Whether dinosaurs get killed or survive the actual impact is irrelevant to both arguments. The issue is whether or not dust from the asteroid impacts set in motion a chain of events that killed the dinosaurs. (C) is a sweeping statement that’s way too broad to be necessary here. Besides, Trent isn’t arguing for many causes, but simply a cause other than asteroid impact. (D) Trent doesn’t need to assume that dust from the impact would entirely fail to cool the climate in order to argue that the impact was not large enough to wipe out the dinosaurs. He may readily admit that such dust has a cooling effect while still maintaining that there simply wasn’t enough of it to have wiped them out. 18. (A) Parallel Reasoning The most efficient way through this Parallel Reasoning question is to recognize the kinds of statements that comprise the original argument, and then to search for the argument that contains not only the same types of statements, but the same number of them as well. Hopefully you recognized the first two sentences as statements of necessity: Paraphrasing, these boil down to “science requires measuring” and “measuring requires selecting units.” These of course can be combined (science requires selecting units), another feature we’ll be looking for in the correct choice. Moving on, let’s hold the conclusion (signaled by “hence”) for last and finish with the evidence: “Selecting units is always arbitrary”—that’s a simple statement of fact—and that brings us to the conclusion that science is therefore arbitrary. So what do we have here? Two linked statements of necessity, a fact and a conclusion. That’s certainly enough to go on, so let’s hit the choices:

Section I: Logical Reasoning

(A) Performing difficult music requires skill, and musical skill requires long practice. There are our two linked statements of necessity, and they can be combined in the same manner as the statements in the original (performing difficult music requires long practice). What about the rest? Long practice is tedious (the statement of fact), so (“hence”) performing difficult music is tedious (the conclusion). All elements present and accounted for. (A)’s perfectly parallel.

not certain languages qualify as foreign and thus satisfy the requirement. (D) is outside the scope. The discussion focuses on the policy of one university, not other universities in North America. 20. (A) Method of Argument

(D) is the closest of the wrong choices. Being a manager requires evaluating, and evaluating entails subjectivity, which isn’t exactly the same as saying it requires subjectivity. We can kill it there if we want. But even the conclusion doesn’t match that of the original. We’d need something in the form of “being a manager is X” to match “science is arbitrary.” Not parallel.

Now to this business of Sedley’s “strategy of argumentation”; this is, as mentioned above, LSAT-speak for a Method of Argument question. This is actually not an uncommon line of reasoning: Sedley tries to show that if Beckstein’s argument holds (if a language shared by many North Americans can’t be used to satisfy the university’s foreign language requirement), French and Spanish would not be able to satisfy the requirement either, based on Beckstein’s criterion. But that, in Sedley’s words, would be “ridiculous.” The elements in (A) match his method perfectly: He’s trying to show that the reasoning (stated above) used to form a conclusion (ASL should not be allowed to satisfy the requirement) would lead to another conclusion (don’t allow French or Spanish either) that’s undesirable (it would be ridiculous to prohibit French or Spanish from satisfying the requirement).

(E) We can eliminate (E) right away because it starts with “some farming.” “Science requires measuring” is far more definitive than “some farming requires irrigation.” Or perhaps you eliminated it because the conclusion states what must happen, which doesn’t match the form of the conclusion in the original.

(B), (D) and (E) Sedley doesn’t question any of Professor Beckstein’s evidence, or even the way he uses it to form his conclusion. Rather Professor Sedley questions the ramifications of Beckstein’s conclusion. All three of these choices incorrectly imply that Professor Sedley has something to say about Professor Beckstein’s evidence.

19. (E) Point at Issue

(C) is clearly wrong: Professor Sedley merely takes issue with Professor Beckstein’s conclusion, not with his authority.

(B) proceeds from only one statement of necessity (running an expanding business requires advertising). No good. (C) “It is permissible…” doesn’t match up with “It is impossible…”, which is to say that the first sentence in (C) doesn’t denote necessity and has no counterpart in the original. We can chop it on that basis alone.

Here’s a two-question stimulus to chew on. The word “disagreeing” in the first stem signals a Point-At-Issue question, to be followed by a Method of Argument challenge in Q. 20. Professor Beckstein argues that because ASL is native to many North Americans, it shouldn’t satisfy the university’s foreign language requirement. Professor Sedley counters that French and Spanish fit the same definition, but it would be ridiculous to exclude them from satisfying the requirement. Both discuss North American students for whom English is not their native language, and whether or not those other languages should be allowed to satisfy the university’s foreign language requirement. This common thread leads us to (E): Beckstein would answer yes, the fact justifies the prohibition; Sedley would say no, it doesn’t. (A) Sedley makes no mention of ASL, so he can’t be disagreeing with Beckstein’s assertion that ASL is the native language of many North Americans. (B) The scope of the argument isn’t “any North American whose native language is not English,” but rather languages that are native to “many North Americans.” (C) Neither professor argues for or against the foreign language requirement itself; they disagree over whether or

21. (C) Assumption Here’s another assumption question, so let’s zero in on the components of the argument. The last sentence (signaled by the Keyword “Hence”) contains the conclusion: Athletes looking to improve muscle strength should not consume engineered foods. The first sentence is largely irrelevant to this conclusion, but the muscle issue comes up in the second sentence, where we learn that the amino acids in engineered foods stimulate growth hormones, which causes growth of connective tissue but not muscle mass. The argument seems to make sense if increasing muscle mass is the only possible benefit from engineered foods—but what if there are other benefits? Wouldn’t it then be reasonable to infer that even athletes who are interested in increasing muscle still might benefit from these foods, and should consider consuming them? This alternative possibility would wreak havoc with the argument, so the author must be assuming that outside of increasing muscle strength, consuming engineered foods confers no benefits at all to athletes. Choice (C) reflects this and gets the point. (A) and (E) have the same problem. The fact is that engineered foods don’t increase muscle mass, and the author concludes on this basis that athletes shouldn’t consume them. The assumption needs to connect these things, which is why the

9

LSAT Practice Test Explained

precise relationship between muscle mass and muscle strength isn’t relevant. (B) Non-athletes are outside the scope of the argument. (D) This issue of obtaining nutrients more easily elsewhere is also outside the scope. This has nothing to do with the argument at hand, which never states or implies just how easy it is to get the stuff found in engineered foods, or whether or not it’s healthier to consume such nutrients in other forms if one can find them. This is all irrelevant, so nothing about this needs to be assumed here. 22. (E) Inference This next Inference question involves a scientifically challenging argument. But buried amidst the technical terms is some fairly straightforward formal logic, so hopefully you didn’t let yourself get bogged down in the terminology. Endo (no reason not to shorten this) is a process by which one organism engulfs another and the engulfed organism becomes a functioning part of the whole. Something strange that looks like a cell nucleus (let’s shorten its cumbersome name to nuc) has been found inside a plant (chlor); can you see where this is going? Perhaps endosymbiosis occurred here? What follows requires some interpretation (and that’s, after all, what Inference questions are all about—interpreting and combining): If the nuc was NOT originally part of an organism that got engulfed, then we’d expect one version of a particular gene in its DNA. That’s a simple formal logic statement, albeit one with complicated terms. Nonetheless, the contrapositive applies: If there’s NOT one version of the gene, then the nuc WAS part of something that got engulfed. And we’re told, in fact, that there are TWO versions of the gene, so we know for sure that engulfing took place. As (E) puts it, chlor was formed as a result of endo. (A) Nowhere is it suggested that other organisms don’t contain nucs. Just because this one particular nuc is found in an organism that underwent endosymbiosis doesn’t mean that only organisms that underwent endo has them. (B) We know that the nuc has genes in its DNA, so it has at least some genetic material, presumably from the previous organism it used to belong to before it got eaten. But who says it contains the entire genetic material of that organism? No way to know. (C) is a major distortion. The origins of nucleomorphs aren’t discussed, and besides, in this scenario, chlor is the result of endo, not a victim of it. (D) A nuc is involved in this particular example of endo, but nowhere is it stated or implied that endosymbiosis must always include nucleomorphs. 23. (C) Parallel Reasoning Considering the sheer length of this Parallel Reasoning question, we have to believe that there are some shortcuts

10

possible; after all, there’s simply no time to slog through every word here. And indeed, the Kaplan strategies cut it down to size. First, compare conclusions: The original conclusion, signaled by the trusty Keyword “therefore,” is that finalism must be “more plausible” than previously thought. Scanning the choices, we see that four of them match this structure well enough, but (D) concludes that something “is false” and can be cut for that reason alone. In fact, the phrase “Therefore it is false” is all you need to read of (D). Similarly, the evidence in the original begins with a view that has been “universally rejected.” That’s a totally unqualified statement that’s not matched by (B)’s “Few…deny” or (E)’s notion that sociologists agree that something may be the case. So fairly quickly we can get this thing down to (A) and (C), and we need to consider the final piece of the original to choose among them. A book intends to repeat the universal rejection of the view, but fails, and therefore the author says the view is actually more likely than before. (A) begins fine by describing the rejection of a claim, but then goes on to supply evidence for the rejection, rather than a new source that repeats the rejection as in the original. (C), however, obliges, as it must because it’s all that’s left: A claim about bike frames is no longer believed (finalism is universally rejected); a new person comes along and makes the same case, but badly (the book launches another attack but fails as a critique); therefore the claim about bike frames that we just heard no bike engineer believes is more believable (finalism is more plausible than people thought). (C) it is. 24. (D) Weaken the Argument Another Weaken question, so as always we paraphrase the argument looking for familiar structures, gaps in the reasoning, and alternative possibilities. The conclusion is that consumers shouldn’t object to manufacturers using old model numbers on upgraded appliances. The reason is that while some people complain that they can’t tell if the model they’re about to buy is the exact model they’re interested in, all modifications are improvements over a previous model. So why should consumers worry about the practice? Well, what if it causes confusion between the old and newer models? What if someone wants a new and improved model she heard of but is sold an older model with the same model name? If (D) is true, this would be a legitimate concern, and the conclusion that consumers have little reason to object to the practice would be weakened. (A) The useful life of an appliance is outside the scope of the argument. (B) could only strengthen the argument, since a more detailed model number would help consumers to distinguish the different variations of appliances. (C) The scope of the argument is the practice of giving different products identical names, not giving identical

Section I: Logical Reasoning

products different names. So this has no bearing on the argument. (E) provides a possible reason why appliance manufacturers engage in the practice, but sheds no light on the issue of whether or not consumers should object to it. 25. (B) Flaw The stem tells us the argument is flawed, and even tells us the source of the problem: an unjustified assumption. So we’ll bring all of our strategies for Flaw and Assumption questions to bear on the problem. The author concludes that there are a lot of planets able to sustain life, based on the fact that one out of the nine planets in our solar system sustains life, and there are a huge number of planetary systems in the universe. If, similar to our system, roughly one planet in each other system can sustain life, then this makes sense—but what if this is not the case? What if the same ratio doesn’t obtain out there in the far reaches of the universe? Then the conclusion would be unsupported. For the conclusion to stand, our solar system must be similar to many other systems (at least in this respect), and no evidence is provided to that effect. (A) The one in nine ratio, not the specific qualities of Earth, serves as the evidence for the claim that lots of planets must be able to sustain life. If there are planets like Earth out there, it’s not necessary that they be the ones to support life. The author doesn’t argue about the specific types of planets that would support life, but rather the number that would based simply on the ratio cited. (C) The argument doesn’t depend at all on our understanding of the conditions necessary for life to arise. What matters is whether or not life can be sustained on a planet. (D) Similar to (A), the author is not at all concerned with the specifics of life on Earth, the conditions that helped it arise, what life-supporting conditions would be like on other planets, etc. He’s focused purely on the numbers, so nothing regarding Earth-like “conditions” is presumed here.

is to get employees to believe they’re contributing, essentially in order to get them to work better (read: harder), while implementing “the ideas that employers want to try.” Inherent in this scheme is the notion that employees will implement new ideas more quickly and efficiently if they perceive themselves to be a positive part of the process, instead of just having orders handed down to them. (B) supports this in every way: Employees are more likely to implement new ideas on improving efficiency that they think they helped to generate. (A) “Employees are more likely to accept suggestions that aren’t obviously directed at them?” Are they being indirectly exposed to subliminal messages (be efficient…be efficient…)? This is out of nowhere. (C), on first inspection, looks almost identical to correct choice (B); the first half starts off well by talking about how employees would respond to ideas better if they were involved in a dialogue with employers. But it goes astray in stating that they’d be less hyped up about ideas generating from a dialogue in which they didn’t participate—no such other dialogue is mentioned. The sociologist says nothing about employers having a dialogue without employees; the choice is implementing ideas derived from employers, or feeling as though they’re part of the dialogue. So (C) veers off at the very end. (D) The sociologist suggests that employees are more likely to implement good ideas if they’re not simply handed down to them, not that they’ll necessarily generate good ideas if they’re happier about the process. Notice how even when they’re brought into the discussion, the intention is still that they’ll accept and implement with greater enthusiasm the ideas employers want to try, so this principle provides no justification. (E) also distorts the situation: It’s the employees who do the implementing, not the employers.

(E) Closer than (D): At least this one focuses on the relevant issue, the numbers. But the author is never this precise about the actual number of planets per system; his argument doesn’t require that most systems have exactly nine planets. He’s interested in the one in nine ratio, which is something entirely different. 26. (B) Principle Finally, we need to find the principle that helps the argument, which means we’re looking for a strengthener stated in general terms. According to the sociologist, employees don’t like to be told flat-out by their employers how to improve efficiency. Instead, an employer should have a more personable chat with an employee, in which both can contribute to the discussion. This will secure employee buy-in, so to speak, and will result in better implementation of the ideas employers want to try. This is actually a masterpiece of corporate manipulation: The trick

11

LSAT Practice Test Explained

Section II: Logic Games

Jackson and Lauder appear, necessarily. So the notation should account for both possibilities:

J

Game 1: Saxophonist Auditions Situation: Saxophonists auditioning. Entities: Six saxophonists. Action: Schedule each saxophonist for an hour-long audition, all on one day. Limitations: Each saxophonist is assigned only one audition that will last for an hour. Every saxophonist auditions. Each audition is scheduled to start every hour between the hours of 1 p.m. and 6 p.m. The Initial Setup: Since we’re sequencing the saxophonists’ auditions, set up a row of six slots left to right. Underneath each slot, list the designated time.

2

3

4

5

Big Deductions: As with any sequencing game, pay close attention to entities working together (Blocks of Entities) as well as entities that appear in more than one rule (Duplications). Start with Rules 1 and 4 since they both give information about Jackson’s relationship to other saxophonists. Rule 1’s limitation can be incorporated with the two possibilities in Rule 4, so write down:

6

The Rules: Rule 1 states that Jackson auditions earlier than Herman. However, it is not known how much earlier Jackson auditions before Herman. So, use an ellipsis between this block of entities:

J...H Rule 2 is similar to Rule 1, although it pertains to different entities. Rule 2 says that Gabrieli auditions earlier than King, but without indicating how much earlier. So, write it out the same way as Rule 1:

G...K Rule 3 creates a tighter bloc of entities than Rules 1 and 2. Gabrieli will audition immediately before OR immediately after Lauder. So, in every possible order is seen either:

GL or LG Rule 4, like Rule 3, creates a tighter bloc of entities than Rules 1 and 2. Rule 4 states that Jackson and Lauder are separated by exactly one audition. But, it is not known in what order

J

Before making deductions, take note that Fujimura is never mentioned in any of the rules. Fujimura is the floater of the game and is not directly restricted from any position.

J

A brief scan of the rules tells us it’s a strict sequencing game. Rules 3 and 4 clearly indicate a strict relationship between various entities.

12

or

L

F G H J K L 1

L

L H or

L

J...H

Likewise, Rules 2 and 3 can be combined since they both provide information about Gabrieli’s relationship to other saxophonists. If Rule 2’s limitation is incorporated with the two possibilities in Rule 3, the following should be written down:

GL . . . K or LG . . . K As with any sequencing game, also keep track of which entities can go first and which can go last. Here, note that G, J, and L can go first while H and K can go last. Recall that F, as a floater, is unrestricted and can go either first or last.

Section II: Logic Games

3. (C) Could Be True

Final Visualization:

Comfort with the rules will make non-“if” questions a little less intimidating.

F G H J K L 1

2

3

4

5

6

GL . . . K or LG . . . K J

L H or

L

J...H

Questions: 1. (E) Acceptability As you go rule by rule, remember to start with the most concrete rule and eliminate violators to quickly find the correct answer. Either Rule 3 or 4 is the easiest to test first. Starting with Rule 3, eliminate any answer choice that does not have Gabrieli and Lauder immediately next to each other. Choice (A) has Gabrieli and Lauder separated by three saxophonists. Eliminate this answer choice. Moving on to Rule 4, eliminate any answer choices that do not have Jackson and Lauder separated by exactly one saxophonist. Choice (C) can be eliminated since it has two saxophonists between Jackson and Lauder. Quickly scan the answer choices for violators of Rule 1—Jackson must audition before Herman. Eliminate (D). Between choices (B) and (E) that remain, one will likely violate Rule 2—Gabrieli must audition before King. That’s choice (B) which can now be eliminated. That leaves (E) as the only acceptable answer. 2. (B) Must Be True Use your initial deductions and master sketch when answering non-“if” questions. The correct answer choice is not merely possible but rather required in all possible sketches. Before looking at the answer choices, take a brief look at your master sketch to look for key deductions. You know from combining Rules 2 and 3 earlier that L will always precede K. That leads directly to answer choice (B). Note that you can still quickly find the correct answer by comparing each answer choice to your master sketch. As you evaluate each answer choice, ask yourself whether this answer choice must be true in your master sketch. Looking through the answer choices, only answer (B) must be true.

Based on initial deductions, eliminate answers (D) and (E). Whether Lauder precedes Gabrieli or follows Gabrieli, King will always be after them both. So, the earliest King can go without assessing the impact of Rules 1 and 4 is 3 p.m. So, draw out a new sketch to see if that works. Based on Rule 4, Jackson will have to occupy the 4 p.m. slot and Lauder will occupy the 2 p.m. slot. This puts Gabrieli in the 1 p.m. slot. The 5 p.m. and 6 p.m. slots can be occupied by either Herman and Fujimura. So the sketch looks like this:

G 1

L 2

K 3

J H/F F/H 4 5 6

Since no rules are violated by placing King in the 3 p.m. slot, answer (C) can be selected as the earliest time King’s audition could be scheduled. 4. (C) Completely Determine When you have made strong deductions in the beginning of the game, even rare questions such as these are worth a shot early on. When faced with trial and error, do so strategically. Right off the bat, answer choice (A) should raise an eyebrow; The only thing we know about Herman is that he must be after Jasper. Given how few rules Herman is involved in, it’s unlikely that placing Herman somewhere will determine everyone’s position. But let’s check. Herman being scheduled at 4 p.m. tells us is that the Jasper/Lauder block has to be scheduled at 1 p.m. and 3 p.m., but it doesn’t determine which one is in which of those two spaces. This puts Gabrieli at 2 p.m., and then King and Fujimura are at 5 p.m.. and 6 p.m., but again, it could just as easily be Fujimura at 5 p.m. and King at 6 p.m., so we don’t have enough information to determine everything. Eliminate. The remaining answer choices will have to be tested one by one. Answers (B) and (C) restrict Jackson, who is likely to have a large effect on the audition scheduling, so start there. (Note: You could just as easily start with answers (D) and (E) in this same regard.) According to answer (B), if Jackson is scheduled at 1 p.m., Lauder must be scheduled at 3 p.m. However, Gabrieli can occupy 2 p.m. or 4 p.m. in accordance with Rule 3. Eliminate answer (B). According to answer (C), if Jackson is scheduled at 5 p.m., this forces Lauder into the 3 p.m. slot. Herman will occupy the 6 p.m. slot. Since Gabrieli must come sometime before King, Gabrieli cannot occupy the 4 p.m. slot and must therefore audition at 2 p.m., immediately before Lauder. Likewise, this forces King into the 4 p.m. slot. The only entity left is Fujimura who must occupy the only remaining audition time of 1 p.m.

13

LSAT Practice Test Explained

Thus, each saxophonist is assigned to an audition time, producing the following minisketch:

F 1

G 2

L 3

K 4

J 5

H 6

So, the condition in answer (C) allows you to completely determine the audition times for each saxophonist, thereby making (C) the correct answer choice. For the record: According to answer (D), if Lauder goes first, that will force Gabrieli into the 2 p.m. position and Jackson into the 3 p.m. position. That still leaves open which positions Herman and Fujimura occupy. Eliminate answer (D). According to answer (E), if Lauder auditions at 2 p.m., that will force Jackson into the 4 p.m. slot. But, Gabrieli can occupy 1 p.m. or 3 p.m. in accordance with Rule 3. Eliminate answer (E). 5. (A) “If” Clause/Could Be True Always consult the rules as you go, incorporating any new “if” clause into your master sketch. This question states that Fujimura cannot audition at 1 p.m. So, this leaves two questions: (1) When can Fujimura audition and (2) Who else can audition at 1 p.m. Since Fujimura is a floater, it remains unrestricted so no huge deductions can be formed there. However, recall at the beginning the deduction that only Gabrieli, Jackson, and Lauder can go first in addition to Fujimura. That answers one of the questions; unfortunately, no previous work helps here. Keeping that in mind, it’s best to go through the answer choices, eliminating any that violate the rules. Answer choice (A) is the easiest to start with since Herman has a fairly minor restriction: Herman must simply come after Jackson. In light of Rule 4 and knowing which saxophonists can audition at 1 p.m., there are a few options:

G 1

L 2

K/F 3

L 1

G 2

J 3

J 1

J 4

F/K H 5 6

K/F F/K H 4 5 6

G/F L G/K/F K/F H 2 3 4 5 6

They all work. This makes answer (A) the correct answer choice. For the record, neither (B), (C), (D), nor (E) are possible because they all force Fujimura into the 1 p.m. slot.

14

6. (E) Must Be True Big deductions made in the beginning of the game can pay off with quick points at the end of the question set. When faced with a Must Be True question without an “if” clause, consider your initial deductions. Early on, it was deduced that Fujimura, Gabrieli, Jackson, or Lauder would have to audition at 1 p.m. while only Fujimura, Herman, or King could audition at 6 p.m. A brief scan of the answer choices should lead to answer (E). It is already known that Lauder can never be later than 5 p.m., so all that is needed is to test out whether Lauder must audition before 5 p.m. The simplest way to do so is to assign Lauder the 5 p.m. audition and build in the rules from there. So the sketch should look like so:

H/F F/H J 1 2 3

G 4

L 5

K 6

Unfortunately, this violates Rule 1: Jackson must precede Herman. Thus, Lauder must always audition before 5 p.m. making (E) the correct answer choice. For the record, the remaining choices can all be disproven if the saxophonists are assigned to the time slot in the answer choice and/or previous work is used. Assigning Gabrieli to the 5 p.m. slot as answer (A) prohibits would not violate any rules. Eliminate. Assigning Herman to the 2 p.m. audition as answer (B) prohibits would not violate any rules and would yield the following sketch:

J 1

H 2

L 3

G K/F F/K 4 5 6

Likewise, this sketch gets rid of answer (D) as well. Finally, answer (C) can be eliminated based on the correct answer to the previous question above.

Section II: Logic Games

Game 2: Furniture Moving

If GSOF → If HSOF or TAB →

Situation: People helping move furniture. Entities: Four people and three pieces of furniture. Action: Assign people in pairs to move each piece of furniture. This is a straightforward Matching game where at least one entity will repeat.

Rule 2 is another formal logic statement and a simple if-then, at that. Write out the original and contrapositive statement like so:

Limitations: We know we will have six slots to fill: two slots for each piece of furniture. We also know that each person will be matched up at least once. So, we will have at least one person or possibly two people assigned more than once. The Initial Setup: With the numbers so definite, this game lends itself nicely to a list/column sketch. The sketch should be anchored around the three pieces of furniture, which will never change number or kind. Two slots should then be drawn under each piece of furniture for each pair of movers. So, the initial sketch should look like so:

REC

SOF

TAB

HREC GREC or TAB

If JTAB → If MSOF or TAB →

MREC JREC or SOF

Rule 3 prohibits Grace and Josh from being paired up to move any piece of furniture. Note this off to the side as

NEVER GJ Big Deductions: When a majority of the rules are conditional statements, you need to ensure that you have correctly contraposed the statements and can keep a keen eye on how to combine those statements. In this game, nothing definitively places any person with any piece of furniture. So be ready to let the new information in the questions lead to key deductions while going through each question. So, the Final Visualization is fairly open:

The Rules:

REC

As you go through each rule, look for limitations in two respects. First, are there any limits on who can move which piece of furniture? Second, are there any limits on who can move furniture together?

SOF

TAB

Rule 1 is a formal logic statement that presents two rules in one.

NEVER GJ

The first part requires that Grace move the sofa if Heather moves the recliner. Regardless of where “if” appears in a sentence, when it appears alone, the “if” will precede the sufficient condition. (Note: There will be an exception to this when the next part of Rule 1 is translated). The contrapositive prevents Grace from moving the sofa if Heather does not move the recliner. Remember to always turn negatives into positives. So, write down this part of the rule like so:

If HREC → If GREC or TAB →

If HREC → If GREC or TAB → If GSOF → If HSOF or TAB → If JTAB → If MSOF or TAB →

GSOF HSOF or TAB

The second part of the rule requires that Grace move the sofa only if Heather moves the recliner. Recall that “only if” precedes the necessary result in a formal logic statement. The contrapositive prevents Grace from moving the sofa if Heather does not move the recliner. So, write down this part of the rule like so:

GSOF HSOF or TAB HREC GREC or TAB MREC JREC or SOF

Questions: 7. (A) Acceptability Even with an open-ended game, Acceptability questions can be good for a relatively easy point. As always, running through the rules one at a time helps eliminate all of the wrong answers. Rule 3 tells us that Grace and Josh can never move furniture together, so that eliminates

15

LSAT Practice Test Explained

(B). Rule 2 tells us that in any assignment where Josh is helping to move the table, Maria must move the recliner. A quick scan of the remaining answer choices leads us to eliminate (C) since Josh is moving the table without Maria moving the recliner. Rule 1 is likely to eliminate the remaining two answer choices since there are two rules in one. (D) can be eliminated since Heather is moving the recliner without Grace moving the sofa. Likewise, eliminate (E) since Grace is moving the sofa without Heather moving the recliner. That leaves (A) as the correct answer. 8. (D) “If” Clause/Must Be True A keen understanding of how formal logic statements and contrapositives can be combined leads to quick points on Test Day. The ability to correctly contrapose your formal logic statements in the rules make this question a snap. If Josh and Maria move the recliner, the contrapositive for Rule 1 kicks in: Heather is NOT moving the recliner, which mandates that Grace NOT move the sofa. If Grace can’t move the sofa or the recliner, but she must move at least one piece of furniture, then Grace must move the table. A quick scan of the answer choices leads to (D)—a perfect match to the big deduction! Even though you might have found a sketch unnecessary, it should look like so:

REC

SOF

J

TAB G

M

10. (B) Could Be True Use your big deductions to narrow down the answer choices quickly. At first blush, it appears that trial and error is necessary. However, Rule 3 removes two answer choices immediately from consideration. Grace and Josh will never be paired up to move furniture so answer (A) can be eliminated. Likewise, if Heather and Maria are paired up to move two pieces of furniture, that will force Grace and Josh to move the third piece of furniture together. Eliminate (D). After strategically narrowing down the answer choices, now you can try out the remaining contenders. Incorporating answer (B)’s pairing will force Heather and Josh to move the sofa together, yielding the following:

REC

SOF

TAB

G

H

G

M

J

M

A quick scan of the rules reveals that the minisketch does not violate any one in particular. Thus, (B) is our correct answer. For the record, answer (C)’s pairing will force Grace and Maria to move the sofa. The problem is with Rule 2: With Josh moving the table, Maria must move the recliner. Eliminate. Answer (E)’s pairing will force both Grace and Heather to only move the sofa. This violates Rule 1’s requirement that Heather moves the recliner whenever Grace moves the sofa. Eliminate. 11. (E) “If” Clause/Could Be True With every new “if” question, consider which big deductions you are specifically being tested on.

9. (B) “If” Clause/Could Be True Always be mindful of when the LSAT is testing the key skill of Understanding Formal Logic. For this question, Heather is our repeating furniture mover. So draw this new condition in like so, incorporating Rule 1:

REC

SOF

TAB

H

H

H

J/M

G

M/J

Knowing that Grace only moves the sofa with Heather eliminates answers (A), (C), (D), and (E). That leaves (B) as the correct answer. Alternatively, you can zero in on (B), since it’s the only possible match of person to furniture in our minisketch. As seen above, all the others are incompatible with the new condition in this question.

16

Assigning Josh and Maria to move the sofa will kick in Rule 1’s contrapositive: Grace is NOT moving the sofa. This presents two big deductions: First, since Heather does not move the recliner or the sofa, she must move the table. Thus, we can eliminate choices (A), (B), and (D). In light of Rule 3 (Grace and Josh can never be paired up, thereby eliminating (C)), only choice (E) is possible.

Section II: Logic Games

Game 3: North American Tree Planting Situation: Planting trees in two public parks. Entities: Two public parks and four kinds of trees. Action: For each park, three of the four kinds of trees available must be selected. Of the four kinds of trees available, at least two kinds will be planted more than once among the two parks. This is a Double Selection game. It could also be viewed as a Selection/Matching Hybrid game. Trees are selected from the four types and matched to one or both of the two parks. The fact that a tree type could be duplicated or go unused is a hallmark of Matching. How the game is characterized is less important than being able to understand the rules and build them into a helpful Master Sketch. Limitations: Rule 1 gives a definite assignment of the number of trees to each park. Going through the remaining rules, look for limitations on which trees can be planted in the same park together. The Initial Setup: Combine a standard Matching list/column sketch with the standard Selection roster of entities sketch to yield the following:

Rule 4 gives us a definite assignment of tree variety to Graystone Park. Build this into the master sketch like so:

GRAY

LAND

M

Big Deductions: Revisit Rule 2 to see what can be gleaned out of that information. Since there are only two parks, there are two possibilities with Rule 2: Either Graystone or Landing will have both maples and sycamores. Each option also includes the possibility that both parks will have maples and sycamores planted within. So this sets up Limited Options like so:

Option 1: GRAY

LAND

M S

M O S T GRAY

LAND

Option 2: GRAY

LAND

M

M S

The Rules: Rule 1, as noted above, assigns exactly three trees per park. A quick scan of the rules after Step 1 should have led you to build this rule in to your master sketch. Rule 2 sets up some possibilities. Note this for now and come back to this rule in Step 4.

At least one park: MS

Now, think about Rule 3. In order for oaks to be planted in a park, there must also be tamaracks. So that’s two slots that must be available under each park. Looking at the master sketch, it’s impossible to plant more than three trees in each park. So the park or parks that have both maples and sycamores will NEVER have oaks. So the third tree in that park will always have to be the only remaining variety: tamaracks. Build this in:

Rule 3 is a conditional statement, common to Selection games. Use your Formal Logic skills to translate and contrapose this statement as follows:

If O → T If Not T → Not O

17

LSAT Practice Test Explained

Option 1: GRAY M S T

LAND

(C) states that Landing Park must be planted with tamaracks. Well, since both parks must be planted with tamaracks, this must be true making (C) the correct answer.

T

Option 2:

For the record, both (D) and (E) are possible, but not necessary. Eliminate both. 14. (A) “If” Clause/Could Be True

GRAY

LAND

M

M S T

T

Both (A) and (B) require that each park be assigned a tree variety that is not entirely clear. On Must Be True questions, possible is not enough. Eliminate both.

In games with limited options, incorporate the new condition into each option. The wrong answers will not be possible in either option. For this question, both parks are planted with sycamores. Draw the options as such:

Option 1: If O → T If Not T → Not O Think through this a bit more. Knowing that every park that has oaks planted in it will also have tamaracks, and that every park that doesn’t have oaks planted in it will always have the remaining three varieties of trees, it can be deduced that each park must have tamaracks planted in it. So with our big deductions, our Final Visualization yields the above two options. Questions: 12. (D) Acceptability When you make big deductions at the beginning of the game, use them in narrowing down to the correct answer choice in the Acceptability question. Before even looking at the answer choices there are two very concrete deductions: Graystone Park is planted with maples and both parks must have tamaracks. That eliminates answers (A), (C), and (E). Looking to Rules 2 and 3, (B) can be eliminated because neither park contains both maples and sycamores. That leaves choice (D) as the correct answer. 13. (C) Must Be True When limited options are set up, remember to search for the answer choice that must be true in both options. We know Graystone must have maples and both parks must have tamaracks. Going through the answer choices, keep these big deductions in mind.

18

GRAY

LAND

M S T

S M/O T

Option 2: GRAY

LAND

M S T

M S T

Compare each answer choice to the options, eliminating the answer choices that are impossible in either option. For (A) to be true, look to Option 1 since it’s the only option where having oaks planted in either park is possible. The sketch for Option 1 should now look like so:

GRAY

LAND

M S T

S O T

Here, one maple tree variety is planted in one park and one oak tree variety is planted in another. Thus, (A) is possible and the correct answer.

Section II: Logic Games

For the record, since both parks will have sycamores, it is impossible to have more maples than sycamores planted. This eliminates (B). Likewise, it is also then impossible to have more oaks than sycamores planted. This eliminates (C). (D) is impossible in both options as Graystone only has the tree varieties of maples, sycamores, and tamaracks. Likewise, Landing only has the option of maples or oaks in Option 1, never both, which eliminates (E). 15. (A) Must Be False Scan for the “odd choice out” to find the only impossible answer choice. Remember, in limited options, it must be impossible in both options to be correct. Use your process of elimination skills and check each answer against the limited options. Looking at both options, maples, sycamores, and tamaracks will be the three trees planted in at least one of the parks. So, it is impossible for both parks to have oaks, as (A) says, in either option. Thus, (A) is the correct answer choice. Even if you didn’t get the limited options in the beginning, you can still use your deductive reasoning skills to answer this question quickly and efficiently by combining Rules 2 and 3. If at least one park is planted with sycamores and maples AND any park planted with oaks must be planted with tamaracks, then it’s impossible for the park planted with sycamores and maples to have oaks, too, as that would require a fourth tree: tamaracks. Thus, at least one park will never have oaks. (A) is impossible. 16. (E) Could Be True Use your limited options in Could Be True questions. All you need is for the answer choice to work in one option and you have found the correct answer choice. Similar to Question 14 above, compare each choice to the options to find the only one that will be possible in at least one option. On Question 15, it was deduced that both parks will never have oaks. Coupled with the knowledge that both parks will have tamaracks, eliminate (A). It is impossible to have an equal number of oaks and tamaracks planted between the two parks. Rule 2 states that at least one park will always have sycamores planted in it. Even if the other park has oaks planted in it, it’s still impossible for oaks to outnumber sycamores between the two parks. Eliminate (B). From the earlier big deduction, you know that both parks will have tamaracks. Eliminate (C) and (D). That leaves (E) as the only possible answer choice. For the record, you can also use your previous work from Question 14’s Option 2 minisketch to determine that (E) is possible.

19

LSAT Practice Test Explained

Game 4: Executive Site Visits Situation: Executives visiting their company’s manufacturing plant sites on different days.

executives. Come back to this once more information is gathered. For now, note as such:

FARM: Exactly 1 EXEC

Entities: Five executives and three sites. Action: Each executive must be scheduled for a site visit among three days. Since each executive will be assigned exactly once and the sites will need to be scheduled on different days, this is a Sequencing/Distribution Hybrid game.

R

Limitations: Each executive will be assigned to exactly one site and each site will be assigned at least one executive. What is not known is exactly how many executives visit each site and in what order the sites are visited.

Q

The Initial Setup: Keep track of your minimums and allow for the “outliers” to be incorporated into your sketch. As with any Hybrid game, your goal is always to get both actions incorporated into a single sketch. This game is no exception and yields itself nicely to this objective.

Rule 4 can be tricky if you rush through it. Sasada’s visit cannot take place after Vandercar’s site visit. However, nothing prevents Sasada from visiting the same site as Vandercar. Note both possibilities as such:

Anchor your sketch around the different days—Day 1, Day 2, and Day 3—then place two rows below each day: the top for the sites and the bottom for the visiting executives:

Q R S T V FARM HOME MORN 1 2 3 SITE: EXEC: (+

)

The Rules: As you go through the rules, be looking for limitations on who can visit which sites, who can visit sites together, and which sites must be visited before others. In addition, determine which action—Sequencing or Distribution—each rule invokes. Rule 1 affects the Sequencing action in the game, requiring that the Farmington site be visited before the Homestead site. Recall that Blocs of Entities can yield Limited Options so you will want to refer back to this rule in Step 4. For now, note as such:

FARM . . . HOME Rule 2 is immensely helpful in narrowing down our numbers options. Knowing that one site—Farmington—will only have one visiting executive, it can be deduced that the other two will either have two visiting executives each or one site will have one visiting executive while the other has three visiting

20

Rule 3 affects both Sequencing and Distribution actions in the game, but should still be incorporated into one notation as such:

T

SV or S . . . V Note that every entity has a condition placed on it so there are no floaters. Big Deductions: As stated earlier, think about how our Bloc of Entities in Rule 1 can set up Limited Options. Farmington can never be last; only Homestead or Morningside can be the last site visited. However, Farmington can be either the first or second site visited and still satisfy Rule 1’s requirement. Break this out into two options. Now think about Rule 2’s number breakdown. Place one slot under Farmington in each option, still being flexible with the two outlier entities to yield the Final Visualization:

Section II: Logic Games

18. (A) “If” Clause/Must Be True

Option 1: Q R S T V 1 (NEVER R/T)

2

FARM

MORN/HOME

SITE:

3 (NEVER Q) HOME/MORN

EXEC:

There are two clues in the “if” clause for Question 18 that should enable you to make some quick deductions. First, there are two executives—Rodriguez and Taylor—visiting the second site. Thus, the second site cannot be Farmington. This means you will be dealing strictly with Option 1. Second, since Rodriguez and Taylor are mentioned, draw your attention to Rule 3. Rodriguez and Taylor must follow Quinn’s site visit, so that means Quinn will visit the first site, which is Farmington in Option 1. These deductions should lead to (A) as the correct answer choice.

“ Option 2: Q R S T V 1 (NEVER R/T)

2

MORN

FARM

SITE:

In a Hybrid game action, you will need to combine both actions to quickly answer certain questions correctly.

3 (NEVER Q) HOME

Recall that every rule, though, must be thought about in “if” questions. Rule 4 prohibits Sasada from visiting a site after Vandercar. Coupled with the numbers requirement that at least one executive must visit each site, it can be deduced that Vandercar must visit Day 3’s site, either with Sasada or without. These deductions yield the following sketch:

EXEC:

1 (NEVER R/T)

2

3 (NEVER Q)

SITE:

FARM

MORN/HOME

HOME/MORN

EXEC:

Q

R

V

T

(S)

“ (+

)

R SV or S . . . V

Q T

Note Rules 3 and 4 off to the side, and make some initial observations. Quinn will never visit the site on Day 3 and neither Rodriguez nor Taylor will visit the site on Day 1.

(S)

17. (A) Acceptability

Now, the only things not known are in which order the Homestead and Morningside sites are visited and which site Sasada visits. Compare the answer choices to your minisketch, eliminating any choices that are merely possible and not necessary.

Even the most difficult game will often come with an Acceptability question. Use this question to get comfortable with the rules.

(B) gives simply one possible site Vandercar can visit on Day 3. Eliminate. Likewise, (C) gives only one possible site that Sasada can visit on Day 2 or Day 3. Eliminate.

Rule 2 is easy to test, since it requires that Farmington only has one visiting executive. Answer (E) clearly violates that rule by scheduling both Rodriguez and Sasada for a visit to Farmington, so eliminate that answer. Likewise, Rule 1 is easy to test: Farmington must always be scheduled for a site visit before Homestead. Farmington is scheduled after Homestead in choice (D), so eliminate that answer.

(D) merely gives one site Sasada can visit that week. Eliminate.

Questions:

Continuing on, Quinn must visit a site before both Rodriguez and Taylor, but (C) has Quinn visiting a site either after or with Rodriguez and Taylor; eliminate that. In choice (B), Sasada visits a site after Vandercar, which violates Rule 4. That answer can be eliminated. That leaves (A) as the only answer that doesn’t violate any of the rules.

Finally, the second of the three site visits could include exactly three of the executives or it could include just two—Rodriguez and Taylor. Eliminate (E). 19. (C) “If” Clause/Could Be True Consider the rules before drawing multiple sketches. Keep Rule 2 in your head to determine which option you are working in. From the new information, you know that Farmington will not be visited by Quinn or Sasada, who are now paired up. In addition, knowing that (1) Quinn cannot be

21

LSAT Practice Test Explained

scheduled to visit the site on Day 3 and (2) Quinn must always be followed by Rodriguez and Taylor, allows some more information to be built into the two Limited Options. In addition, think about where Vandercar can/must go and who visits Farmington. Vandercar must either follow Sasada or see the same site as Sasada. Incorporating this information, only Option 2 is possible since Vandercar would be the only executive left to visit Farmington first in Option 1, prohibitively before Sasada:

1 (NEVER R/T)

2

SITE:

MORN

FARM

HOME

EXEC:

S

V

R

Q

3 (NEVER Q)

T

Now, compare each choice to your minisketch, eliminating any that are impossible. Both (A) and (B) can be eliminated because you are strictly in Option 2. So Farmington must be visited on Day 2, which requires Homestead be visited on Day 3. (C) is possible if Vandercar is the only executive to visit Farmington on Day 2. This would force Rodriguez and Taylor into Day 3 in accordance with Rule 3. Thus, (C) is the correct answer.

answer. (Note that Quinn can visit Homestead with Vandercar, but not with both Sasada and Vandercar as (D) points out.) While you wouldn’t want to waste time drawing this all out on Test Day, here is the minisketch to verify that Quinn and Sasada cannot visit Homestead together on Day 2 in Option 1:

1 (NEVER R/T)

2

3 (NEVER Q)

SITE:

FARM

MORN/HOME

HOME/MORN

EXEC:

S

Q

R

V

T

21. (E) “If” Clause/Could Be True In any “could be true,” be careful to assess all possibilities and not make any unwarranted assumptions of where entities must or cannot be assigned. According to this question, Morningside will be visited by both Quinn and Vandercar. We will need to incorporate this information into Options 1 and 2. Recall that Quinn can only be scheduled for Day 1 or Day 2. So in Option 1, Morningside must be scheduled for Day 2, visited by Quinn and Vandercar. Keep Rule 3 in mind, recalling that Rodriguez and Taylor must visit a site after Quinn, which is Homestead in Day 3. That only leaves Sasada to visit Farmington on Day 1:

For the record, (D) and (E) can be eliminated because neither is possible in Option 2 with the new “if” clause incorporated. 20. (D) CANNOT Be True Proper rule interpretation is essential to prevent missing many questions. For this question, the correct answer will be impossible in light of the rules. Since there are Limited Options, each answer choice has to be tested against the options before ruling it out. Using the previous question’s minisketch, eliminate (B) as a possible scheduling of executives to Homestead. For the remaining answer choices, use your key deductions to eliminate any possibilities. It is known that Homestead will always be visited on Day 2 or Day 3. Choices (A) and (D) both include Quinn who can never visit a site on Day 3. Test these answer choices out next since they appear to be restricted to only being possible in Option 1. If Quinn visits Homestead on Day 2 in Option 1, that forces Rodriquez and Taylor into Day 3. Sasada will have to visit Farmington on Day 1, since it must precede Vandercar. So Quinn will never visit Homestead with Sasada. That means (D) is impossible and thus the correct

22

1 (NEVER R/T)

2

3 (NEVER Q)

SITE:

FARM

MORN

HOME

EXEC:

S

Q

R

V

T

A quick scan of answer choices reveals no answer that is possible in our Option 1 minisketch. Do the same with Option 2. Morningside will be visited on Day 1 by Quinn and Vandercar. That leaves Rodriguez and Taylor to visit Farmington and Homestead on Day 2 and Day 3, respectively. Pay close attention to Rule 4. Sasada can never visit a site after Vandercar. So, since Vandercar is visiting Morningside on Day 1, Sasada will have to visit Morningside on Day 1, as well. Since no rules are violated, scan the answer choices for a possibility in our new Option 2 minisketch. Neither (A), (B), (C), nor (D) are possible in the Option 2 minisketch, so (E) is the only possible correct answer choice. To verify:

Section II: Logic Games

1 (NEVER R/T)

2

3 (NEVER Q)

SITE:

MORN

FARM

HOME

EXEC:

Q

R/T

R/T

S

Option 1: 1 (NEVER R/T)

2

3 (NEVER Q)

SITE:

FARM

MORN/HOME

HOME/MORN

EXEC:

S

Q

R

(V)

T

1 (NEVER R/T)

2

3 (NEVER Q)

SITE:

MORN

FARM

HOME

EXEC:

Q

S

V

V

22. (E) Must Be True When a Must Be True question contains no “if” clause, compare the answer choices against each option to determine what must be true in both options. For an answer choice to be correct, it must be true in both Option 1 and Option 2. Compare each one by one. When possible, use your previous work. Option 2 allows for Farmington to be visited after Morningside and on Day 2 by only one executive. Eliminate (A) and (D). Question 17’s correct answer shows Vandercar visiting Morningside on Day 3 after Rodriguez visits Homestead on Day 2. Eliminate (B). Question 18’s minisketch shows that Sasada can visit Morningside or Homestead on Day 3. Eliminate (C). So, (E) is the only possible correct answer choice. 23. (B) “If” Clause/Must Be True Use the clues in the “if” clause to zero in on the rule likely to yield the most deductions.

Option 2:

R T Before checking each answer choice one by one, assess what is common between both options in addition to Sasada visiting Farmington. In either option Rodriguez and Taylor will always be scheduled to visit the site on Day 3. This leads to choice (B). For the record, choices (A), (C), (D), and (E) are possible in either Option 1 or 2, but not required in both options. Eliminate.

This question schedules Sasada to visit Farmington, which can be true in either option. In addition to this new information, also incorporate Rule 4’s requirement that Sasada precedes Vandercar since Farmington can only be visited by one executive—-in this case Sasada. In addition, Rule 3 tells us that Quinn must visit a site on Day 2 or 3. Build this information into the minisketches for both options:

23

LSAT Practice Test Explained

Section III: Logical Reasoning 1. (E) Flaw Whenever a bold claim is made in a Logical Flaw question, look for alternative circumstances the author has overlooked. The author concludes that a shark attack is no more likely for night swimmers than day swimmers based on two pieces of evidence. First, most of the region’s swimmers are day swimmers. Second, all recent shark attacks in the region have occurred during the day, not at night. Here, the author has mistakenly taken evidence about what has recently been the case to then argue that anything else is unlikely. To frame it in the context of the argument, the author mistakenly assumes that just because all recent shark attacks have happened during the day, night attacks are highly unlikely. What the author is overlooking is that an increase in night swimming could increase the likelihood of shark attacks at night. After all, if no one’s in the water at night, the sharks have no one to attack. This prediction leads directly to choice (E). For the record: (A) Irrelevant Comparison. While this may be true, it is not the primary flaw in the argument. Whether sharks prefer to hunt at night or in the daytime does not have any bearing on whether there will be people in the water for them to attack. (B) Out of Scope. The source of the evidence is not only immaterial to the lack of validity of the author’s contention, but also is never mentioned. (C) Irrelevant Comparison. Similar to choice (A), choice (C)’s flaw is immaterial to whether night swimmers will actually stand a greater likelihood of being attacked. (D) Out of Scope. Whether swimmers are the most knowledgeable about the safest time to swim does not directly affect their likelihood of being attacked by a shark. In addition, until the number of night swimmers is comparable to day swimmers, the swimmers may not even be able to have an accurate assessment of the safest time of day to swim. 2. (A) Point at Issue When faced with an unfamiliar question stem, read it carefully and determine exactly what the question is asking for in the correct answer choice. Characterizing answer choices goes a long way. Here, the question stem asks for what Denise and Reshmi agree upon. So this is a variation on a Point at Issue question. Take a moment and characterize the answer choice. The correct answer choice will be a point of agreement. Thus, the four

24

wrong answer choices will be points of disagreement or outside the scope of at least one speaker’s argument. Denise begins by claiming that a reduction in crime requires that punishment be certain and sufficiently severe to serve as a deterrent against those potentially engaging in criminal behavior. Reshmi responds by arguing that crime can only be reduced if educational opportunities are made available to everyone as a means of providing an alternative to the criminal lifestyle. Where they differ is on what is required to reduce crime. Where they agree is that those pondering committing a criminal act will have enough sense to consider the consequences and alternatives to what they are doing. That’s a huge leap. Before either of Denise and Reshmi’s arguments is assessed, it has to be possible that either can have the intended effect of altering an individual’s behavior. That’s choice (A). For the record: (B) Out of Scope. Neither Denise nor Reshmi ever discusses the degree of importance assigned to crime within modern society. (C) Out of Scope. This is Denise’s underlying contention. Reshmi never discusses this claim. (D) Out of Scope. Why crimes are committed is never discussed by either Denise or Reshmi. (E) Out of Scope. Again, this is Denise’s underlying contention to which Reshmi neither responds nor discusses. 3. (B) Weaken the Argument Keywords go a long way to helping seek out the evidence and conclusion. Use this knowledge to then determine a fact that would make the conclusion more or less likely to be true. The conclusion is the first sentence and everything else that follows is evidence. The author makes the contention that Acme Corporation offers great opportunities for advancement to unskilled workers. How can the author make so broad a claim? On the single piece of evidence that Ms. Garon was able to rise to Acme’s presidency from an “entry-level position requiring no special skills” when she first started as an assembly line worker. When the author is supporting a broad or general contention on the basis of a specific example, look for facts that will provide similar examples to strengthen the argument or point out that the specific examples offered as evidence are an anomaly to weaken the argument. In this case, review each answer choice for a fact that points out the latter: (A) provides an additional example of an entry-level worker who was able to rise to a position of status within the company, which makes the author’s contention more likely. This is the exact opposite of what the question is asking for in the correct answer choice. Eliminate. (B) notes that Acme’s common practice is to put individuals with MBA’s into entry-level positions before promoting them

Section III: Logical Reasoning

to management, which goes against the author’s primary contention that Acme is known for promoting unskilled workers to management. Choice (B) is the correct answer choice. For the record: (C) Irrelevant Comparison. This statement could just as easily lend more credence to the author’s contention if it could be determined that Acme’s employees tend to be unskilled. In any event, more information is needed before determining whether this answer choice helps or hurts the author. (D) Outside the Scope. How long Ms. Garon worked at Acme does not have any effect on the author’s contention that she is currently the president, yet started out in an entry-level position as an unskilled worker. (E) Outside the Scope. What Acme pays in comparison to other companies has no effect, good or bad, on the author’s contention that Acme provides excellent advancement opportunities for those starting out as unskilled entry-level workers. 4. (C) Assumption When faced with a lengthy stimulus, be sure to read through with a critical eye; not every piece of evidence will be equally relevant to the author’s conclusion. Here, the author provides a lot of background information on the behaviors that enable yellow warblers to communicate with each other. The contrast Keyword “however” signals a shift in tone. What follows will be the crux of the author’s argument. Solely on the evidence that yellow warblers dare not enter in or around any yellow warbler singing its molting song, the author concludes that yellow warblers have no competition for food supply within this area. This begs the question: Do any animals besides yellow warblers—namely, other birds with the same diet and feeding territory—disregard the yellow warbler’s molting song and feed on the same food supply within the range of the warbler’s restricted fly zone? In order for the author to be completely certain that singing, molting yellow warblers have no competition for their food during this time, the author must be certain that the answer to the above question is no. Choice (C) ensures that the author can be sure that there is no additional competition. For the record: (A) Irrelevant Comparison. Whether the food supply is enough to sustain the yellow warbler is immaterial. What matters is whether only the yellow warbler has access to the food supply. (B) Outside the Scope. Whether warblers in general are the only molting birds that engage in this behavior does not mean that additional competition stays away when they engage in this behavior. (D) Outside the Scope. For the argument to be valid, the proper assumption must concern other kinds of birds that DO

eat the same food supply as the warblers. If anything, this answer goes against the author’s conclusion. (E) Outside the Scope. The size of the feeding territory has no relevance to whether the singing, molting yellow warbler is the only bird to lay claim to it during the warbler’s molting time. 5. (E) Flaw When a Flaw question requires an evaluation of two speakers’ arguments, expect one speaker to point out a flaw in that of the other speaker’s argument. The question stem asks for the flaw in Chinh’s argument according to Lana. Chinh is arguing that TV producers should disregard the preferences of the viewers when making creative decisions. Chinh then presents a seemingly dissimilar situation in support of the contention. Recall that whenever an author uses an analogy for evidence, it must be true that the subject matter of both the analogy and the conclusion are comparable. Here, Lana points out that Chinh’s analogy is indeed not analogous because TV producers “create” for the satisfaction of the viewers. Lana then offers her own analogy to CEOs who must consider consumers’ tastes when developing new products. Lana is pointing out that Chinh’s analogy is flawed and cannot be used as evidence to support Chinh’s conclusion. This prediction matches choice (E). For the record: (A) Outside the Scope. This type of argumentative strategy, though flawed, is inappropriate as applied to Chinh’s argument. Rather than seeking to support his conclusion about TV producers by claiming that TV producers do not consider the viewers’ preference, Chinh offers additional evidence. In any event, this is not the flaw that Lana points out. (B) Outside the Scope. The representativeness of the consumer sample in Lana’s argument, or lack thereof, is not at issue. Neither is Lana’s argument. (C) Outside the Scope. This answer choice is the same flaw as choice (A), just worded differently. Again, it’s not the flaw that Lana points out. (D) Irrelevant Comparison. Lana does not even respond to Chinh’s analogy regarding painters disregard for the interest of the museum-going public. So this answer choice would be immaterial to her issue with Chinh’s argument. 6. (E) Assumption When a term appears in the relevant piece of evidence, but nowhere in the conclusion, be sure to sync this up in the Assumption. The author concludes that individuals who are more susceptible to heart disease as a result of high sodium intake should eat fresh fruit and vegetables rather than canned or frozen fruit and vegetables. The author bases her conclusion on the single piece of evidence that potassium in plant foods—such as

25

LSAT Practice Test Explained

fruits and vegetables—prevents sodium’s bad effects. Since the author is concluding that fresh fruits and vegetables are likely to help the individuals in question more so than canned or frozen fruits and vegetables, it must be true that potassium appears in higher quantities in fresh rather than canned or frozen fruits and vegetables. Choice (E) matches this prediction. For the record:

(D) and (E) attempt to assign intent to inanimate objects: heavy rain and a power plant. This does not match the original argument, which concerns a human actor, Dana.

(A), (B), and (C) Irrelevant Comparison. Each of these answer choices compares potassium to sodium content in order to show that more potassium than sodium is present in various situations. Such may be the case, but the assumption needs to shore up that potassium is contained in fresh fruits and vegetables more so than canned or frozen fruits and vegetables, regardless of the sodium concentration.

The author has a fairly tenuous conclusion. The author states that a glacier probably deposited the boulder 100 miles south of its birthplace. In order to weaken this conclusion, the correct answer choice will have to contribute to eliminating ANY possibility—even one aberration—that this could have happened. Evaluate the answer choices looking for a fact that does just that.

(D) Outside the Scope. There are two things wrong with this answer choice. First, the argument tells us the benefits of potassium. The number of negative side effects is immaterial. Second, there’s no comparison between fresh versus. canned and frozen fruits and vegetables so it cannot support the author’s conclusion that fresh is better than the other.

(A)’s “Most” is not good enough. It leaves open the possibility that the author could still be correct. Eliminate.

7. (C) Parallel Reasoning (Flaw) In Parallel Flaw questions, disregard any answer choices that do not contain the same kind of flaw contained in the original argument. The correct answer choice must contain the same kind of evidence, the same kind of conclusion, and the same kind of flaw as the original argument. It is best to take the specific content and make it abstract to facilitate the locating of the correct answer choice. The author concludes that Dana intentionally killed the plant on the basis of knowing two things. First, Dana intentionally watered the plant every other day. Second, the frequent watering killed the plant. The flaw here is assuming that the actor (Dana) had the intended consequence (the death of the plant) from the action (frequent watering). Without more information, the author cannot assume that Dana meant to kill the plant. Assess the answer choice with this prediction in mind: (A) does not match the structure of the argument. While Jack intended to steal the money from Kelly, the resulting effect did not have a detrimental effect on Kelly nor does the argument in choice (A) conclude that Jack intended to steal $100 from Kelly. Eliminate. (B) has no flaw. It follows logically that if Celeste knows that coffee is grown in Peru then she knows that coffee is grown in South America. Eliminate. (C) is a match. The argument faultily concludes that the actor (the restaurant owner) had the intended consequence (the disappointment of Jerry) from the action (removing the dish from the menu). For the record:

26

8. (D) Weaken the Argument The strength of the wording in the correct answer choice must match the strength of the wording in the stimulus.

(B) leaves open the possibility that the glacier could have moved the boulder 100 miles south. Even a slim possibility is still a possibility. Eliminate (C) actually strengthens the argument by presenting additional evidence of volcanic rock north of the deposited boulder. If the closest source of volcanic rock is 50 miles north, who’s to say there’s not more 100 miles north? Eliminate (D) does the job. If there are NO geological sources of volcanic rock north of this border, then it is HIGHLY unlikely that the boulder was NOT deposited by the glacier 100 miles south of its origin. Remember, the answer choice need only contribute to the impossibility. For the record: (E) Outside the Scope. Knowing that no volcanic rock exists within 50 miles of the border does not eliminate the possibility that it exists 100 miles north of the boulder’s resting place. 9. (B) Method of Argument Formal Logic Keywords go a long way in helping you understand the logic of the argument. Here, the presence of the word “unless” should have alerted you to how Rifka is supporting her conclusion that they need not stop for directions: IF they needed to stop for directions → they would be lost. Craig reverses or denies Rifka’s reasoning to conclude the exact opposite: IF we are lost → we need to stop for directions. So the answer choice needs to show that Craig is denying the truth of Rifka’s evidence to show that they need to in fact do the exact opposite of what Rifka is concluding. Choice (B) is a match. For the record: (A), (C), and (E) 180. Each answer choice says that Craig agrees or does not take issue with Rifka’s reasoning, which is the opposite of what we’re looking for. (D) Outside the Scope. At no point does Craig present a counterexample nor does Rifka make a generalization. Rifka’s

Section III: Logical Reasoning

argument pertains to her and Craig only. Craig is reasoning the exact opposite way from Rifka and not bringing in any outside information.

(A) Frank is not arguing that Lance is employing circular reasoning. Rather than argue that a general rule has at least one exception simply because this is the case, Lance actually bases his claim on a conditional truth. Eliminate.

10. (D) Assumption

(B) is a match. Frank is clearly showing that Lance has contradicted himself, which is exactly why he is demanding that Lance withdraw his conclusion. For the record:

When faced with lengthy and complex stimuli, focus on the most relevant portions of the argument. The author begins by noting how romance literature differs from satirical literature: romance presents an idealized world while satire presents a debased world. Then, the author points out how the two genres are similar: the main characters both have moral qualities reflective of their respective worlds. Finally, the author switches gears to discussing comedy and tragedy. Signaled by the conclusion Keyword “therefore,” the author states that neither comedy nor tragedy can be classified within either of these two genres—romance and satire literature—simply because comedy and tragedy require that the moral qualities of their main characters change within the course of the story. What’s missing is that the moral qualities of the main characters in romance and satire never change. For if they do change, then the conclusion cannot be true. Even a quick scan of the answer choices leads right to (D)—an exact match to the predicted assumption. For the record: (A) Outside the Scope. The argument strictly focuses on main characters. Moreover, even if this were true, it still does not give the conclusion any more support than it already had. (B) Outside the Scope. The visions of the world as within comedy and tragedy are never discussed in the stimulus so it cannot have any effect on the conclusion. (C) Irrelevant Comparison. How the main character’s moral qualities change throughout the course of a tragedy is immaterial to whether the same happens in romance or satire. (E) Irrelevant Comparison. This answer does nothing more than restate the evidence, a classic wrong answer trap in Assumption questions. If it’s mentioned in the stimulus, it’s not the missing piece of the argument. 11. (B) Method of Argument Insight into the common argument methods that appear on the LSAT assures a quick point. First, evaluate Lance’s argument. Lance maintains that experience causes us to know that every general rule has at least one exception. Frank then characterizes Lance’s conclusion itself as a general rule. On that basis alone, Frank points out the invalidity of Lance’s conclusion as a conditional statement to which there is no exception—a violation of Lance’s exact statement. Frank is taking Lance’s argument to show that his general principle cannot be true. This is a common LSAT argumentative technique. Evaluate each answer choice for a correct description of Frank’s rhetorical moves:

(C) Outside the Scope. Frank never argues that general rules can or cannot have exceptions. Frank simply takes Lance’s argument and shows that it is a contradiction in itself. (D)Outside the Scope. Frank never mentions experience or responds to Lance’s mention of experience. Moreover, Frank does not invoke anything other than Lance’s own reasoning to show that Lance is wrong. (E) Outside the Scope. Frank never assesses the validity of Lance’s statement in relation to real cases. He need not since, even in a general sense, Lance’s conclusion cannot be true. 12. (E) Flaw Beware of the author’s overlooked alternatives—the most common type of Logical Flaw on the LSAT. The author concludes that the energy subsidy has failed to achieve its intended purpose simply because many of the most isolated rural populations still have no electricity, even after energy production was subsidized to rural residents in the hopes of increasing their access to electricity. However, in a Flaw question, the argument must be assessed with a critical eye. The author overlooks the possibility that there may be other rural residents who benefited from the subsidy, perhaps even many, who do not live in the most isolated areas. Until the author confirms that most of the rural residents in general have no better access to electricity than they did before the subsidy took effect, the author cannot validly conclude that the subsidy has not achieved its purpose. This predicted flaw matches choice (E)’s possibility that the subsidy actually helped many other rural residents in the nation. For the record: (A) Outside the Scope. The author’s Scope is limited to what was achieved after the subsidy went into effect. Any other circumstance is not relevant to assess the validity of the author’s conclusion. (B) Outside the Scope. The author is strictly discussing the effect the subsidy had on the intended beneficiaries, not those who may have received gains indirectly. (C) Outside the Scope. Any group outside the rural residents that were intended to benefit from the subsidy is irrelevant to the argument. (D) Outside the Scope. Whether the urban area residents benefited or not from the subsidy is irrelevant to the effect it had on the rural residents.

27

LSAT Practice Test Explained

13. (A) Paradox When a standard question type appears, jump on it for a quick-and-easy point. The stimulus presents a puzzling series of statements. Heart attacks likely occur on Mondays because that’s when people have to go back to work, a high anxiety-inducing event. So, then how can it be true that unemployed retirees are more likely to have heart attacks on Mondays than on other days? The correct answer will resolve this mystery and present a reason that unemployed retirees could still be experiencing more heart attacks on Mondays. Evaluate each answer choice with this prediction in mind: (A) Bingo. Even though they are no longer working, the unemployed retired people still associate Monday with work and experience a comparable amount of stress with beginning large projects on this day. For the record: (B), (C), (D), and (E) can all be eliminated because they give no reason why unemployed retirees are more likely to experience heart attacks on Monday rather than any other day of the week. Choice (A) contains a link to Monday, specifically. 14. (D) Strengthen the Argument In order to support an author’s conclusion regarding a study, eliminate any inconsistencies or variances that could give rise to an unrepresentative sample. The conclusion, signaled by the Keywords “This indicates that,” holds that overconfident persons are more likely to start a business than less confident people in the face of seemingly insurmountable odds. The author bases her belief on a survey of 100 entrepreneurs and 100 business managers, which resulted in the entrepreneurs being much more confident in their survey answers than the business managers. To link overconfidence with an inclination to start up a business, the likelihood of starting up a business must be more closely associated to the level of confidence as opposed to a person’s chosen trade. Simply put, for the conclusion to hold, whether one is an entrepreneur, a business manager, or a zookeeper, the most overconfident will exhibit a more noticeable pattern of starting up businesses. Choice (D) does just this by attributing the same inclination to business managers solely based on their high level of confidence. For the record: (A) Irrelevant Comparison. The nature of the questions is not at issue. Rather, the author is primarily concerned with how confident the participants were in their responses. (B) Outside the Scope. Whether the business’ success was foreseeable to the entrepreneurs does not matter. It’s the inclination to start the business and its link to overconfidence in the first place that must be solidified. An “accurate determination of the odds” is too far a stretch to link to overconfidence. If anything, this would make it seem that

28

the entrepreneur’s confidence level was justified and not necessarily overconfident, at all. (C) Outside the Scope. It’s not the correlation between confident and success that the author is concerned with here. Rather, it is the correlation between confidence and the likelihood of starting a business. (E) Outside the Scope. The correlation between confidence and business savvy is not discussed by the author. The correlation between confidence and the likelihood of starting a business matters to the author regardless of how strategic or on point that start-up ends up being. 15. (B) Flaw When a Flaw question has Formal Logic, look for the author to be mistaking necessity for sufficiency. This is a classic Formal Logic question testing the ability to correctly translate the statements. The author gives us three sets of conditional statements to translate. The first translates as IF Agnes’ proposal is approved → the 4th FL lab will be cleaned out. Note that the second sentence about Immanuel’s use of less space is a red herring. Focus on the Formal Logic statements. The second condition maintains that IF Immanuel’s proposal is approved → Immanuel will continue to use the 2nd FL lab. The third condition translates as Only Y are X or IF the director approves any proposal → the director supports the proposal. On these three statements, the author mistakenly concludes that since the director supports both proposals, the director will approve both proposals, including Agnes’, which means the 4th FL lab will be cleaned out. The author has made the classic necessity versus sufficiency flaw of reversing without negating. Specifically, the author is reversing the logic of the third statement to mean that IF director supports proposal → director approves proposal. That’s the only way to know that because if the director supports Agnes’ proposal, then the director will approve Agnes’ proposal. Just because the director will only approve the proposals she supports does not mean she will approve any proposal she supports. This exact flaw is stated in choice (B). For the record: (A)Faulty Use of Detail. Disregarding the second sentence above guards against too much focus being paid to the detail in the second sentence. As a result, this choice can be easily rejected. The size of the labs is immaterial. (C) Outside the Scope. The level of enthusiasm with which the director supports the proposal is of no relevance to determining how the author goes wrong in this argument. (D) Outside the Scope. Immanuel’s desires after the proposals are approved are immaterial. The author is strictly concerned with the direct effect the approvals will have.

Section III: Logical Reasoning

(E) Outside the Scope. The adequacy of the lab is of no consequence to determining whether Agnes’ proposal will be approved.

opinion on this matter? Do they disagree on this matter? The correct answer choice will answer all three of those questions in the affirmative.

16. (D) Principle

(A) Glen never speaks to citizens’ ability to make good choices in light of governmental interference. Eliminate.

Stay focused on the Scope of the applicable principle. Wrong answer choices are likely to be Outside the Scope.

(B)Glen never assesses the protection of citizen’s rights. Eliminate.

The author concludes that Outdoor Sports Company’s decision to provide incentives to existing customers who give their friends’ emails is an unethical business practice. The author says so because the incentives encourage people to exploit their personal relationships for financial gain, an act that calls into question the integrity of those personal relationships. To determine the broad rule, the correct answer choice will link up an unethical business practice as one that encourages people to risk damaging the integrity of those relationships. This exact principle is stated in choice (D). For the record:

(C)Glen never speaks to the inherent danger of allowing government to decide what constitutes virtuous behavior. Eliminate.

(A) Distortion. This answer puts too much emphasis on exploiting the relationships for profit. Any behavior that has the intended effect of damaging the stability of the personal relationships is considered unethical. Choice (A) is too narrow to be an applicable principle. (B)Outside the Scope. The principle is based on the effect the practice has on encouraging participation, not the use of the information that is then given over by the participant. (C) Outside the Scope. The principle’s unethical distinction is dependent on an indirect effect, regardless of the practitioner’s intended consequence. (E)Outside the Scope. Whether the damage to the personal relationship results from the practice is immaterial to the principle’s assessment that the practice is unethical. The practice need only encourage the behavior, whether or not the relationships are actually damaged. 17. (E) Point at Issue First, assess each speaker’s argument. Then, use the Kaplan Decision Tree to narrow in on the correct answer choice. Glen maintains that law’s primary role should be to create virtuous citizens, on the basis that an emphasis on law’s purely procedural side leads an individual to be more concerned with themselves than others. There’s a Scope shift here from what the emphasis on law leads to versus what makes a virtuous citizen. In response, Sara argues that if the law’s sole concern was creating virtuous citizens, then the government would have more control over people’s lives. Sara notes that this danger is far worse than the government’s focus on protecting individual rights. Rather than predict an answer, go through your Kaplan Point at Issue Decision Tree, interrogating each answer choice with three questions: Does Glen have an opinion on this matter? Does Sara have an

(D) Sara never speaks to the result from an emphasis on law’s purely procedural side. Eliminate. (E) is the only choice left and must be correct. Glen argues that law’s primary role should be to create virtuous citizens. Since Sara argues that this would lead to something dangerous and argues in favor of another primary focus of law—to protect individuals’ rights at all costs—then choice (E)’s issue must be what they disagree on. 18. (C) Principle When a Principle question mimics a Parallel Reasoning question, the correct answer choice will exemplify the same kind of principle as in the question stem. We’ve all seen the ads “Buy now! Make no payments for six months!” Well, here the author concludes that even though some credit card companies allow cardholders to skip payments, it’s almost never in the cardholder’s interest to do so. Primarily, the author argues, because the finance charges keep accumulating and the cardholder ends up paying much more than she would have six months earlier. So, the underlying principle is that even a current decision appearing to have a great benefit can have negative consequences in the future. Evaluate each answer choice to determine whether this principle is illustrated. (A) The author never states that cardholders should find other ways to achieve the same benefit. Eliminate. (B)The author points out that the offer actually increases the amount of money owed to the credit card companies, not the amount of money the credit card companies must pay. Eliminate. (C) A perfect match. Just like the offer in the stimulus above, which carries heavy finance charges later on, the immediate benefit of using funds for new roads will have greater maintenance costs in the long run. Choice (C) is correct. For the record: (D) Irrelevant Comparison. There is no comparison between new versus used in the original argument. (E) The practicality or use of the benefit is not at issue. The long-term costs of the offer are at issue in the original stimulus.

29

LSAT Practice Test Explained

19. (A) Inference Use the ability to translate complex or difficult Formal Logic statements when faced with tough questions towards the end of the section. This question contains one of the most complex Formal Logic statements: No X is Y. But, both instances of this statement can be translated easily along with the second sentence to yield deductions. Move swiftly, translating each statement and its contrapositive. Recall that “No X is Y” means that the members of group X and the members of group Y are exclusive. So in this case, the first sentence translates as “IF literature student → NOT physics student.” Likewise, the contrapositive is “IF physics student → NOT literature student.” Skip the second statement for now and translate the third statement as “IF rhetoric student → NOT physics student,” with the contrapositive “IF physics student → NOT rhetoric student.” Now, turn attention to the second statement. Several students taking physics are taking art. How many remains unclear. But even if one physics student is taking art, it must be true that the student is not taking rhetoric and not taking literature. On this deduction, choice (A) must be true. There are at least some students (several, in fact) who are taking art but not literature—because we know that they are physics students. Voila! For the record: (B) is unwarranted. The connecting link is physics. The art students must be taking physics, also, to be sure they are not taking literature. Without this link, it is possible that some literature students are taking art.

(A) Outside the Scope. The appropriateness of psychotherapists’ attempts to entertain a broad audience is immaterial. It’s the quality of professional help being administered in the situations that guides the psychologists’ belief. (B) Extreme. Although the psychologist disagrees with any psychotherapists’ decision to give advice on talk shows, the psychologist never goes so far as to claim that it is the desire to entertain a broad audience that matters more than the nature of the advice being given. (C)Distortion. This is tricky. The psychologist is making a claim regarding the appropriateness of the behavior of the psychotherapists while using an indirect reference to the quality of the advice that would be provided on talk shows. To say that psychotherapy should never be provided when there is even the slightest chance of compromise with quality is different from the psychologists feeling that psychotherapy should not be provided when it is “nearly always incompatible with providing high-quality psychological help.” (D) Outside the Scope. The interests of the audience are immaterial to the psychologist’s claim. The psychologist is focused on the quality of advice, regardless of whether this whets the audience’s appetite. 21. (B) Point at Issue

(C) Same problem as choice (B). Until it is known that those same students are also taking physics, we cannot be sure that they are not also taking rhetoric.

Point at Issue questions containing Formal Logic should be a welcome relief towards the end of the section since you have two fail-proof strategies to get the correct answer choice.

(D)The author provides no connection between literature and rhetoric that would exclude students from taking both at the same time. Rather, the author merely states that any physics student cannot take either literature or rhetoric.

Here, the correct answer choice will point out the matter on which Tania and Monique disagree. Tania is arguing that art critics cannot give a fair critique because they cannot separate themselves from the emotional and passionate connections art requires. In fact, Tania points out that in order to give a truly unbiased opinion, a good art critic would have to be speaking about things of no interest to him. Monique goes on to point out that a good art critic can still be fully engaged in the artwork, yet render an unbiased opinion after consulting general principles of aesthetics. Note that neither is arguing whether art is passionate. Tania and Monique are arguing over what makes a good art critic: unbiased? Or passionate? Once you’ve elicited out the issue, evaluate each answer choice for a match to the prediction.

(E) This could be true, as long as these students are not taking physics. However, without this key piece of information, choice (E) is not a must be true and must be eliminated. 20. (E) Principle When a Principle question mimics an Assumption question, invoke the Kaplan strategy for Assumption questions to find the underlying principle. The question stem resembles an Assumption question. The conclusion can be easily located using the Keywords “For this reason.” The psychologist is concluding that psychotherapists should never provide psychotherapy on talk shows since the desire to entertain is almost always incompatible with providing high-quality professional help to the guests on the talk shows. So the psychologist believes that psychotherapy should only be administered in a manner likely to be of high

30

quality. Anything else is unacceptable. Although stated in the negative, this prediction is a match for choice (E). For the record:

(A) Tania does not argue that art is simply a passion, just that it is a passion. Eliminate. (B) This is a match to our prediction. Tania thinks a good art critic is incapable of being unbiased and Monique is arguing that a good art critic is capable of being unbiased. For the record:

Section III: Logical Reasoning

(C) This is a point of agreement. Both Tania and Monique argue that art critics should feel emotion towards artworks. (D) There’s no argument over what defines fairness. They both are using “unbiased” as an appropriate synonym for fairness. (E) Neither Tania nor Monique makes a value judgment as to whether passionate engagement of the art critic is the most important aspect of art criticism. They both seem to agree that an art critic, first and foremost, should be fair. 22. (C) Paradox When dealing with an abstractly worded stimulus, focus on Keywords to draw out the meaning of each statement. The author begins by stating that the writing styles associated with high literary quality works are susceptible to misinterpretation. Then, the author uses that to conclude that judicial decisions interpreting the law are rarely of high literary quality. Next comes the problem: It’s common to find high literary quality in dissenting opinions, which can sometimes be included in the judicial decisions. So, the question is how can it be true that a judicial decision can still be written with a high literary quality yet want to prevent any misinterpretation? Evaluate each answer choice for a statement that resolves this question. (A) actually deepens the mystery by increasing the likelihood that those who are not attributed with a high literary quality of writing would have an even greater influence on the dissenting opinions. Eliminate. (B) does not provide a resolution. In fact, the answer is concerning literary works and legal opinions in general when the author is focusing on works of high literary quality and judicial decisions. Eliminate. (C) resolves the issue at hand. If the law is not determined to any great extent by dissenting opinions, then the judicial decisions can contain dissenting opinions yet still guard against misinterpretation. Thus, choice (C) is the correct answer. For the record: (D) and (E) both present situations that do not touch on either side of the issue. Eliminate since they are Outside the Scope of the paradox. 23. (E) Inference Formal Logic appears all over the LSAT. Know that a strong understanding of this key skill will help you swiftly move through questions that appear later in the section. In the first sentence the ecologist claims that without the intervention of conservationists, the squirrel monkeys will become extinct. In other words, the conservationists’ intervention is required for the squirrel monkeys’ survival. This statement yields the following translation: IF squirrel monkeys survive → conservationists intervene. Likewise, the contrapositive states that IF NO conservationists’ intervention

→ squirrel monkeys extinct. Then the ecologist gives us a condition under which the squirrel monkeys will survive: IF large tracts of habitat preserved → squirrel monkeys survive. The contrapositive then translates as IF squirrel monkeys extinct → large tracts of habitat NOT preserved. The last sentence is just filler and is of no consequence to the logic of the argument. Stay focused on how the first two statements can be connected through common terms as you evaluate each answer choice for a statement that must be true. (A)The ecologist only discusses the habitat in question. The food supply or lack thereof in other habitats is outside the scope and the ecologist does not provide enough information to know that this must be true. (B) This need not necessarily be true. The preservation of the large tract of habitat is one way to ensure survival of the squirrel monkeys. There could be a whole host of other ways that the intervening conservationists can save the squirrel monkeys that the ecologist never discusses. (C) The ecologist states that squirrel monkeys flourish when a plentiful supply of insects and fruit is available. The ecologist never states that the squirrel monkeys would die off without this food supply. Choice (C) is going too far. (D) This answer reverses the logic of the first statement. If the squirrel monkeys survive, it is because the conservationists have intervened. The ecologist never states that intervention alone will guarantee their existence. In fact, the second statement provides an unrelated condition—the preservation of the habitat—that will ensure survival of the squirrel monkeys. (E) This answer must be correct since the other four are demonstrably wrong. This answer emphasizes the connection between the two formal logic statements, but in the form of the contrapositive. IF conservationists’ do NOT intervene → squirrel monkeys extinct → large tracts of land NOT preserved. Choice (E) is correct and properly removes the common term of “squirrel monkeys extinct” in the contrapositive. 24. (A) Strengthen the Argument When faced with a lengthy stimulus late in the section, be that much more diligent about focusing first on the conclusion and then on the relevant evidence. A brief scan of the stimulus hones in on the conclusion Keyword “Thus” in the last sentence. Note the necessity language in the author’s conclusion. The amount of Byzantine documents sealed with lead seals must have exceeded the 40,000 lead seals that remain today. How is the author so sure that the 40,000 seals were not enough? Well, the purpose of the lead seals back then was to seal the document until it was opened. Once the document had been opened, the author concludes that the lead seals were most likely recycled

31

LSAT Practice Test Explained

since they cost so much. In order for this to be more likely, there must have been more documents to which the recycled lead seals were affixed that were being opened during that time. Otherwise, the author cannot be sure that 40,000 lead seals were not enough to cover demand. This prediction leads to choice (A). For the record:

(A) and (D) can be eliminated on the basis that the farmer only has one conclusion in the argument. There is neither an intermediary nor main conclusion.

(B) differs just enough from choice (A) to kill it as a viable answer choice. It’s not whether the documents were destroyed since the destruction would not have required a lead seal. It’s whether the documents were opened—that’s what would have required the lead seals to be continuously manufactured.

(E)can be eliminated because the statement in question, if anything, is a consequence of the phenomenon—the insects’ growing resistance—rather than the phenomenon itself.

(C) presents an irrelevant comparison. The amount of lead available versus now or then is immaterial since the author has already determined that the lead seals were recycled in order to meet the demand. (D) contradicts the author’s conclusion. If there were at most 40,000 documents then the 40,000 lead seals would have been enough to meet the demand, thereby undermining the author’s conclusion. (E) still leaves open the possibility that the 40,000 seals may have been enough. Especially if they were recast after each use. 25. (B) Role of a Statement When assessing what role a statement plays in an argument, first identify whether it is conclusion or evidence. The question is asking for how the farmer uses the statement “farmers have to use greater and greater amounts of costly insecticides to control insect pests.” In the absence of any conclusion Keywords in the stimulus, the one-sentence test will help isolate the components of the farmer’s argument. Right off the bat, the first sentence appears to be the farmer’s opinion and main point: “It is counterproductive for farmers to use insecticides.” This is not the statement the question is asking about, but this opening sentence is the primary conclusion in the farmer’s argument. So, it is likely that the statement in the question stem will be used as evidence to support this conclusion. Sure enough, the statement is found in the next sentence, which begins with the evidence Keyword “because.” The farmer is supporting his contention that the use of insecticides is counterproductive since the more insecticides farmers use, the more resistance the insects build up to the insecticides. So, the correct answer choice must affirm that the statement in question is used as support for the main conclusion. The only answer choice that fits the prediction is choice (B). The farmer gives us the causal explanation for why farmers use greater and greater amounts of insecticides: the insects build up a resistance to the insecticides. This supports the overall conclusion that the use of insecticides is counterproductive for farmers. For the record:

32

(C) can be eliminated since the statement in question is not the conclusion but rather is used as evidence to support the farmer’s contention.

Section IV: Logical Reasoning

Section IV: Logical Reasoning 1. (C) Flaw Being wrong about one thing doesn’t make someone wrong all the time. Anna starts off this discussion by presenting information about rainbows that was claimed by scholar Pliny the Elder. William’s problem with the rainbow claim is that Pliny the Elder made other claims about the existence of headless people and curing headaches with snails—seemingly dubious claims to say the least. However, what William fails to consider is that the truth of those claims have no bearing on the truth of the rainbow claims. Pliny the Elder could have been wrong about the headless people, but still correct about the rainbows. Choice (C) points out William’s error in suggesting that making invalid claims on one topic invalidates your claims on another topic. For the record: (A) Distortion. William doesn’t make Anna’s conclusion appear extreme; he merely suggests that it’s wrong. (B) Irrelevant Comparison. While William may assume that Pliny the Elder was in bad faith when he reported about the strange creatures, that’s not the problem with his argument. The flaw is that he assumes Pliny the Elder was in bad faith when he reported about the rainbows. (D) 180. William clearly seems to find Pliny the Elder’s assertions to be absurd and invalid. (E) Irrelevant Comparison. The timing of Pliny the Elder’s writing is irrelevant to the argument. Pliny the Elder could have written his claims about headless people last year and William would probably still make the same argument. 2. (E) Inference Watch out for extreme language in the answer choices. The shareholder has some concerns. Despite the success of the company’s current operations, the company is looking to expand to the food service industry. However, that move comes with a high risk and the potential of siphoning money from other operations. The shareholder finishes by pointing out that a second option for expansion—pharmaceuticals—would entail a lower risk. So, that simply suggests that expansion into pharmaceuticals would likely result in less chance of losing money than expansion into food service—a sentiment perfectly stated in (E). For the record: (A) Outside the Scope. Actually, the shareholder claims that the current operations are a time-proven success. There’s no indication of a need for more funding.

(B)Extreme. We know that expansion into pharmaceuticals would provide a lower risk, but that doesn’t guarantee that siphoning would be avoided altogether. (C) Extreme. Note that the shareholder claims that the food services operation may siphon funds from other operations and simply comes with a high risk. That kind of wording suggests that losing money is definitely possible, but not a certainty. (D) Extreme. Similar to (C), this answer suggests that expansion into food services will result in a loss. However, even a high-risk prospect could still result in a profit, so the company could still make money in the food service industry. Additionally, the extreme use of the word “only” suggests that the company couldn’t be profitable in any other scenario. 3. (B) Principle When a question asks about one of two opinions in a stimulus, watch out for answers that validate the wrong opinion. In this argument, Mariah concludes that Adam should not judge the essay contest. However, unlike Joanna (who suggests bias, which Mariah refutes due to the anonymity of the contestants), Mariah’s reasoning is that Adam has no experience in critiquing essays. So, the principle behind Mariah’s argument is that a lack of experience constitutes a reason to be dismissed as a judge. (B) is the only principle that uses that criterion. For the record: (A) Distortion. This is tempting, since Mariah does reject Joanna’s suggestion of bias. However, Mariah doesn’t reject this reasoning because she feels that bias is insufficient grounds for dismissal. She rejects it because the essay authors are anonymous, which suggests that there wouldn’t be bias in this case. (C), (D) 180. On the contrary, Mariah is suggesting that expertise is the main factor for dismissing Adam as a judge. (E) Outside the Scope. This might be a principle that Joanna would use, but Mariah’s argument is about expertise, not bias. 4. (A) Flaw Confusing properties of a whole with the properties of its individual parts is a classic flaw on the LSAT. In this argument, we learn that product NoSmoke is made up of two ingredients. A study showed that one of those ingredients by itself did not decrease cigarette cravings. The author then concludes that, if the same is true for the second ingredient, then NoSmoke as a whole would not decrease cravings. However, the author ignores the possibility that, although each ingredient is ineffective by itself, the two ingredients may decrease cravings when combined. (A) nicely points out the author’s assumption that whatever can’t be done by the

33

LSAT Practice Test Explained

individual ingredients can’t be done by the full product with the ingredients combined. For the record: (B) 180. There is no correlation in the argument and the author is claiming a lack of causation. (C) Outside the Scope. The study used smokers and the conclusion was about reducing craving in smokers, so there’s no reason to suspect that the study is unrepresentative. (D) Outside the Scope. This argument is not concerned with quitting smoking. It’s only about whether or not NoSmoke reduces craving. (E) Outside the Scope. The author never states or implies any claim of bias. 5. (D) Main Point In Main Point questions, words of contrast frequently indicate the author’s disagreement with previous statements. The gardener starts off by introducing a recommendation made by researchers: allow some weeds to grow to repel caterpillars. However, once the gardener starts the second sentence with the word “while,” we get the sense that the gardener has a bone of contention with the researchers. Sure enough, the gardener calls the recommendation “premature.” This surely seems like the gardener’s Main Point. What follows this claim is simply evidence to back up the gardener’s dissent. So, simply put, the gardener’s point is that the researcher’s recommendation to allow weeds to go is premature. (D) is a good summarization of this cautionary opinion. For the record: (A) Irrelevant Comparison. The gardener does claim that it’s wise to avoid using pesticides, but this is not the primary concern at hand. The gardener is far more concerned about the recommendation regarding letting weeds grow. (B) 180. The researchers may believe this, but the last sentence suggests a much bleaker picture in the eyes of the gardener. (C), (E) Extreme. The gardener never offers an opinion on whether or not letting weeds grow would actually repel caterpillars. The gardener is more concerned about whether or not this practice should be done. 6. (D) Method of Argument In Method of Argument questions, stay focused on the structure of the argument rather than the content. In this argument, the executive provides two situations: ads were printed in a magazine, and ads were posted on that magazine’s website. The executive then mentions that results were only available for one of the two situations: online. Without having data from the print ads, the executive simply concludes that the results of the print ads would probably mirror the results of the online ads. So, lacking data about one situation,

34

the executive makes a conclusion about that situation by using existing data from a second, similar situation—which is pretty much what (D) says. For the record: (A) Outside the Scope. The author’s prediction of the response to the print ads is based on information from the online ads, not to any factor that caused the print ads. (B) Outside the Scope. Actually, the author uses information from the online ads—a situation with results that are more limited than the typical results. (C) Extreme. The only specific instance we have information about is the online advertisement. One is hardly a large number of specific instances. (E) Outside the Scope. Close, but the executive’s conclusion is about the current performance of the print ads, not the future performance of those ads. 7. (E) Paradox Watch out for answers that verify or strengthen the facts, but don’t resolve the mystery at hand. Something unusual has happened here. Coyotes have been removed from a small island because they were preying on wild cats and, more importantly, the plovers that were dwindling in number. One would expect that removing their predators (the coyotes) would help the plovers’ population rebound. Instead, once the coyotes were gone, the plover numbers plummeted and they disappeared altogether. Why would that have happened? The key here is that the coyotes also preyed on wild cats (a piece of information that should not have gone unnoticed). If, as (E) says, the wild cats also preyed on plovers, the poor plovers merely had one predator replaced with an increase in another predator. That explains why the plovers didn’t survive, and thus answers the question. For the record: (A) Irrelevant Comparison. This may provide evidence as to why coyotes prey on plovers, but it doesn’t resolve the issue as to why the plovers disappeared when the coyotes were taken away. (B) Outside the Scope. Without knowing what happened to the wild cat population after the coyotes were removed, this answer can’t help explain the plovers’ situation. Furthermore, we’d probably expect the wild cat population to increase after their predators, the coyotes, were gone. If the plovers fluctuated along with the wild cats, we would expect them both to have increased, thus furthering the mystery rather than explaining it. (C) Irrelevant Comparison. Whether the coyotes were susceptible to a certain disease is irrelevant to the disappearance of the plovers. And since the disease is considered common to plovers, this wouldn’t explain the dramatic turn of events as described in the stimulus.

Section IV: Logical Reasoning

(D) Outside the Scope. Having lived on the island for an unspecified amount of time with coyotes as predators, it would certainly be understandable if the wild cat population wasn’t as big as it used to be. However, this has no bearing on the plight of the plovers.

Furthermore, if, as (C) says, bookstores are now larger, that could weaken the argument by showing how consumers now have more books to choose from. And if, as (D) says, bookstores are now smaller, that could also weaken the argument by showing that smaller stores can help consumers find what they’re looking for more efficiently.

8. (A) Strengthen the Argument

(E) 180. This also weakens the author’s argument by suggesting that the variety of books is not as important to consumers as price. Without knowing if chain stores are more expensive, this answer doesn’t factor into the author’s argument.

Many arguments on the LSAT focus on only one factor in making a decision and ignore other potentially more important factors. The economist here states that there are two solutions to cutting personnel costs: lay off employees or reduce wages. The economist then concludes that companies are likely to lay off employees for the sole reason that layoffs are less damaging to morale—as if morale was the most important consideration in making the decision. To strengthen this view, we want an answer that verifies the economist’s assumption that companies make this decision based on its effect on morale. That’s exactly what (A) does. For the record: (B) Outside the Scope. This talks about when companies would increase wages, a concept that is never discussed in this argument. (C) Outside the Scope. The argument is not about whether companies can make a profit, but how they can reduce costs. (D) Outside the Scope. This may explain what would happen if companies chose the reduced wages route, but doesn’t strengthen the author’s point that layoffs are the likely route. Furthermore, if reduced wages did lead to resignations, one could argue (perhaps a little cynically) that companies would be more likely to choose this route instead as it would lead to a potentially greater reduction in personnel costs. (E) 180. This actually goes against the author by providing a reason why companies would not want to lay off employees.

10. (C) Flaw If an argument has more than one possible flaw, then beware of answers that distort one of those flaws or take it to extremes. Contrary to the critics, the concert promoter is suggesting that a particular concert series does have popular appeal. The evidence is merely that sales of memorabilia are equal to those of other, comparable concert series. This works fine if the sale of memorabilia indicates popular appeal. That seems to match answer (B), but the word “sole” seems a little off. The promoter merely assumes that memorabilia sales are one indicator, not necessarily the only indicator of popular appeal. That word should have sent up a red flag. Eliminate. Thankfully, the author makes another mistake in reasoning. The promoter concludes that the one series has popular appeal because of its similarity to other series. However, we’re never told if those other series have popular appeal. If those other series were unpopular, then having equivalent sales would probably indicate a lack of popularity in the promoter’s series. So, the promoter also errs by assuming the popularity of the comparable series, a flaw that is accurately (and not extremely) presented in (C). For the record:

9. (A) Assumption

(A) Sale figures are facts, not emotional considerations.

Bridging the unmatched terms in the evidence and conclusion can lead to solid predictions.

(D) The promoter actually states that the other events are “comparable,” not dissimilar.

Here, the author is concluding that the success of chain bookstores is a detriment to book consumers. Why? Because this success has led to a damper on the variety of available books. Simply put, the author is assuming that a lack of variety is a detriment to consumers. In other words, as (A) puts it, consumers would be better off with more variety. For the record:

(E) While the author doesn’t make this distinction, this is not a flaw. The argument is entirely concerned with the quality of the series, not of the individual components of the series.

(B) 180. This weakens the argument by showing a benefit to chain stores, making their success seem more positive and less detrimental.

We’re given a little bit of information about two types of UV radiation from the sun: UV-A (which causes wrinkles) and UV-B (which causes sunburn). We’re also told that sunscreens used before ten years ago protected users from UV-B and not UV-A. Combining the information we know, that means that sunscreens used before ten years ago protected users from sunburn, but not from wrinkles. So, wrinkles were just as

(C), (D) Irrelevant Comparison. While exact opposite answers might suggest that one is right and the other is wrong, here the concept of bookstore size is irrelevant to the author’s argument regarding variety—making both answers wrong.

11. (E) Inference Stick to the information given and beware of making faulty assumptions.

35

LSAT Practice Test Explained

likely to occur with the older sunscreen as without, which is what (E) says. For the record: (A) Outside the Scope. The stimulus only tells us about the change in sunscreen protection, not in how often sunscreen is used. (B)Irrelevant Comparison. Since older sunscreen didn’t protect people from wrinkles, there may be plenty of people who have premature wrinkles and did wear sunscreen. Also, people could still use new sunscreen and get premature wrinkles from something other than UV-A radiation. (C) Outside the Scope. Just because the sunscreen didn’t protect users from UV-A radiation doesn’t mean the cause of premature wrinkling was unknown. Sunscreen makers just may have been unable to create a sunscreen that protects users from UV-A radiation until ten years ago. (D)Extreme. The statements imply that newer sunscreen has UV-A protection. However, that does not mean that it no longer has UV-B protection. Newer sunscreen may protect users from both types of UV radiation, meaning that users are still as protected from sunburn as they were in the past. 12. (A) Principle Strong language in an argument’s conclusion often indicates a conclusion that incorporates more than the evidence indicates. The advice columnist starts by presenting data that shows how people with stress are more likely to be seriously injured playing competitive sports than people without stress. Because of this, the columnist goes one step further and suggests that people with stress shouldn’t participate in any sports—competitive or otherwise. The columnist appears to be assuming that if people with stress should avoid a specific type of activity (in this case, competitive sports), then they should also avoid all activities in the same general category (in this case, sports in general). That principle is outlined in (A). For the record: (B) Extreme. The columnist’s recommendation is about a method that shouldn’t be used. We don’t know if methods that should be used must be subject to scientific study, too. (C) Extreme. Recommending something for a group of people does not mean recommending the opposite for another group. So, just because the columnist recommends avoiding competitive sports to stressed people, that doesn’t mean the columnist recommends competing in competitive sports to nonstressed people. (D) 180. If the columnist makes the jump from avoiding competitive sports to avoiding all sports, it would seem just as likely that the columnist would make the jump from avoiding competitive sports to avoiding any competitive activity. This principle seems quite contrary to the columnist’s reasoning.

36

(E) FUD. Probably not a bad rule of thumb, and probably something the columnist would endorse. However, this focuses too much on people with a history of sports injuries while the columnist is concerned with anyone coping with stress—regardless of injury history. 13. (A) Strengthen the Argument Don’t let scientific words throw you. Paraphrase the argument in simple terms. The conclusion of this argument uses the term “communal foraging.” Instead of rooting through your knowledge of ecology, you should see that the argument defines this for you quite neatly at the end. In basic terms, the argument concludes that tent caterpillars let their nestmates know where food can be found. The evidence for this is that tent caterpillars leave behind a trail of chemicals when looking for food, and they leave stronger trails when they find food sources and weaker trails when they don’t. To validate this argument, we want something that would verify that the stronger trail is used for the purpose of giving information to the other members of the nest. (A) This works perfectly. If hungry caterpillars are more likely to follow trails of chemicals, then that indicates they are using the strength of the trail to find a definite source of food rather than wasting time looking around. That strengthens the idea that the trails are left for “communal foraging,” as defined in the argument. For the record: (B) If caterpillars can’t detect the concentration of pheromones, then strong and weak trails would seem identical. In that case, leaving a strong trail wouldn’t seem to help the nestmates find food any faster. (C) This may provide a little insight into how a caterpillar lives, but it doesn’t connect the chemical trails to providing information about food sources. (D) This is nice to know, but it doesn’t connect the different strengths of the trails to the concept of communal foraging. (E) It’s important that the nestmates are able to detect the chemicals, in order to interpret information about food sources. However, knowing that other species of caterpillars can detect the chemicals is irrelevant. 14. (B) Parallel Reasoning (Flaw) It’s helpful to spot classic LSAT flaws in Parallel Reasoning questions. The argument in the stimulus follows a flawed pattern of reasoning that the LSAT loves to test. The argument states that movies with top actors tend to do well. It then concludes that movies with unknown actors will not do well. This is a classic example of treating a characteristic of a group as if the characteristic were unique to that group. It’s like saying “dogs have four legs, therefore animals that aren’t dogs can’t

Section IV: Logical Reasoning

have four legs.” Recognizing the flaw, we need to find the one answer that has the exact same flaw in logic.

problem. Even history writers who emphasize historical trends still probably study details to spot those trends.

(A) This argument tells us that animals need to devote most of their energy to finding food to maintain optimal energy levels. It then concludes that devoting energy elsewhere will result in less than optimal energy levels. There is no flaw in this logic. Eliminate.

(B) Outside the Scope. Like (A), historical trends can still be discerned when emphasizing details. The history writers just choose not to emphasize those trends.

(B) Here, we are told that gardens with bee balm have the eventual characteristic of abundant crops. Then the argument concludes that gardens without bee balm will not have abundant crops. That’s the same flaw as the original argument, making this the correct answer. For the record: (C) This argument says that keeping confidences is important to friendship because keeping confidences allows for openness. The argument then goes a little too far in claiming that openness is therefore essential to friendship. While slightly flawed in its own right, it’s not the same flaw as the original argument. (D) This is close, but not quite right. Here, visual aids bring about the characteristic of allowing visual conceptualization. However, the conclusion doesn’t say that without visual aids you can’t have visual conceptualization. It just says that conceptualization will be more difficult. This would be more like the original argument if the original argument concluded that movies with unknown actors would have a harder time doing well. Close may be close, but it’s still not right. (E)This argument terrifically illustrates the classic LSAT flaw of confusing necessity with sufficiency. The argument states that understanding rules of perspective is necessary for success, but then concludes that understanding rules will lead to success. A necessary condition is not sufficient to guarantee a definite result. A great flaw to recognize, but not the same flaw as the original argument. Therefore, this is not the correct answer. 15. (D) Inference Watch out for answers that use the exact words from the stimulus but distort the author’s meaning. For this question, we learn of a trend in history writing: An emphasis on historical trends is being replaced by an emphasis on details. As a result, the author states that historical trends are overlooked. As a further result, history writers will also overlook parallels between historic trends and current trends, and this lessens our ability to learn. So, combining all of this information, the author is saying that this new trend (the change from emphasis on trends to emphasis on details) inevitably results in a lessening of our ability to learn. That’s exactly what (D) says. For the record: (A) Outside the Scope. Studying details isn’t the problem. It’s putting emphasis on those details instead of trends that’s the

(C) Distortion. A tempting answer, but distorted in a number of ways. What lessens our ability to learn is overlooking the parallels between historical and current trends. Just looking at historical trends doesn’t necessarily result in the best ability to learn. Also, this answer says people who look at historical trends and not details are best able to learn. Maybe the people who learn best are those who look at both. In short: Since the statements just identify what lessens our ability to learn, we can’t truly infer what makes our ability to learn the best. (E) Extreme. The author might go for this, but it’s just as likely that the author would want a return to emphasizing only historical trends. The author never makes a clear indication either way, so this is not supported. 16. (A) Assumption When a question has a strong Formal Logic component, expect a wrong answer to reverse or inverse the logic. The therapist starts off with a claim: Trust is essential to happiness. Using formal logic, this translates to: If you have happiness, then you must have trust. Or, by the contrapositive: If you don’t have trust, then you can’t be happy. The evidence to back this up is a string of formal logic: If you don’t have trust, then you have no meaningful emotional connection; and if you have no meaningful emotional connection, then you feel isolated. This is all good, but the formal logic in the evidence starts with a lack of trust and ends at feeling isolated. The conclusion, on the other hand, says that a lack of trust leads to no happiness. The therapist never makes the connection from isolation to unhappiness. That’s the assumption, as stated in (A). For the record: (B) Extreme. The therapist claims that a lack of trust leads to no emotional connection. It can be validly claimed that the therapist also assumes that anyone who has no emotional connections will be unhappy. However, that does not mean that anyone who does have emotional connections will be happy. That inverts the formal logic and doesn’t help connect the evidence to the conclusion. (C) Irrelevant Comparison. This is perfectly accurate based on the evidence. In fact, it’s the contrapositive of the evidence. However, it fails to connect that evidence to the concept of happiness in the conclusion, so it doesn’t work as an assumption. Remember that an assumption is an unstated piece of information that connects the evidence to

37

LSAT Practice Test Explained

the conclusion. This answer restates the evidence in other words—a classic LSAT trap. (D) Outside the Scope. The assumption is about people who do feel isolated. Adding information about people do don’t feel isolated is irrelevant to the argument. (E) Irrelevant Comparison. This answer reverses the formal logic in the evidence, but it still doesn’t connect the evidence to the concept of happiness in the conclusion. 17. (C) Flaw Be on the lookout for common flaws that appear consistently on the LSAT. The author presents us with two pieces of information: Sirat Bani Hilal is the only Arabic epic poem still performed today and it’s usually sung (unlike most of the other epics). It’s an interesting correlation, but the author then makes the sudden leap that the singing factor is the primary reason for the epic’s continued performance. That’s a textbook example of an author implying causation based on a mere correlation. (C) points out this often used LSAT flaw. For the record: (A)Extreme. None of the evidence here seems impossible (or even that difficult) to corroborate. (B) Outside the Scope. The fact that the play is still performed and the fact that it’s sung do not indicate biased information.

(B) Distortion. A very tempting answer choice, but problematic. This answer only tells us about charities that have increased their donations. However, what about all of the charities that haven’t increased donations? The fund-raiser is trying to show that increasing emotional connections leads to increased donations. This answer too conveniently ignores the possibility that a lot of charities also increased emotional connections but failed to increase donations. By reversing the cause and effect, this answer distorts the assumption and is therefore incorrect. (C) Irrelevant Comparison. This answer strengthens the evidence about increasing emotional connections, but fails to connect that evidence to the concept of increased donations mentioned in the conclusion. (D) Extreme. The fund-raiser may feel that reducing a donor’s influence may lead to a decrease in donations. However, the fund-raiser never goes so far as to suggest that donors would stop giving any money to the charity. Furthermore, the right to vote is described as a direct influence on policies, but the fund-raiser never suggests what would happen if donors didn’t have any influence whatsoever.

(D) Extreme. The only opinion offered in this argument is the author’s. Without knowing what the public opinion is, we can’t say the author is turning opinion into fact.

19. (E) Main Point

(E) Distortion. The evidence is merely a collection of facts. There are no sufficient or necessary conditions for the author to confuse.

From the very first sentence, Leslie’s main conclusion is pretty much clear: Erich’s quest for the treasure is irrational. By saying “I’ll show you,” Leslie is indicating that everything from that point forward will pretty much be evidence to back up that point. And sure enough, by the time Leslie states “I rest my case,” the point has been made: The quest for the treasure is irrational. That’s answer (E). For the record:

18. (E) Assumption Recognizing a Scope shift from evidence to conclusion can help quickly eliminate wordy wrong answers. The fund-raiser has some concerns about donors to charity. By denying them a vote on an organization’s policy, the fund-raiser suggests that donors will feel less emotionally connected to the charity. This suggests that giving donors the right to vote will give them a greater emotional connection to the charity. However, the author then makes the jump to say that giving donors the right to vote will increase the amount of money donors will give. There’s a distinction between having an emotional connection and giving money that the fund-raiser doesn’t account for. The fund-raiser makes the assumption that a greater emotional connection will lead to larger monetary donations. (E) captures the essence of that assumption. For the record: (A) Extreme. There are many problems with this answer, the most important of which is that it offers no connection to why the author feels that a right to vote will lead to increased

38

donations. Furthermore, a right to vote doesn’t have to be the most effective way for donors to influence the charity’s decisions. As long as it’s a way, donors will probably feel more emotionally connected.

Some wrong answers to Main Point questions will restate facts from the argument that act merely as evidence.

(A) FUD. This is true. However, Leslie was just using that analogical example to prove the point about Erich’s quest for the treasure. It’s a point, but not the Main Point. (B) Extreme. Leslie clearly doesn’t want Erich to risk his physical well-being for this particular treasure (which Leslie claims is less valuable than the world). But this answer says “regardless of the possible gains,” which eliminates the possibility of risking physical well-being for things far more valuable than the treasure in question. Leslie never goes that far, so this answer doesn’t work. (C) Extreme. Close, but Leslie doesn’t claim the treasure has no value—just that it’s less valuable than the world. (D) Distortion. Leslie may hope that Erich is convinced, but Leslie’s argument is directed at telling Erich that the quest is irrational. Use the last line to clarify this point. When Leslie says “I rest my case,” this is Leslie saying “Your quest is irrational.”

Section IV: Logical Reasoning

It’s not Leslie saying “You can be convinced that your quest is irrational.” That subtle distinction is what makes (D) wrong and (E) correct.

(A) Irrelevant Comparison. There’s no need for the people learning dances in 1995 to be the same people learning dances now. People could be dancing now because they heard how much fun other people were having.

20. (D) Weaken the Argument

(B) Outside the Scope. George doesn’t want to know why it was unpopular before 1995; he wants to know why it is popular now.

A number of arguments that show the benefit of an action can be weakened by showing unmentioned negative effects. The article in question here refers to a study showing that people who take vitamin C supplements tend to have a lower risk of heart disease. Based on this study, the article concludes that people who take vitamin C supplements are overall healthier. It’s hard to accept an argument about one’s overall health when that argument focuses solely on heart disease. To weaken this argument, the correct answer will reference some non-heart-related health factor that these supplements tend to affect negatively. (D) does the trick by showing how the supplements reduce one’s resistance to infectious diseases. For the record: (A) Irrelevant Comparison. The argument is about the health benefits of vitamin C supplements. The effects of vitamin C outside of supplements have no bearing on the argument, especially since we’re not told whether the supplements are better or worse—just different. (B) Outside the Scope. This answer says that other dietary changes would reduce heart disease just as well as vitamin C supplements, but that doesn’t change that fact that vitamin C supplements still reduce heart disease. Therefore, this has no effect on the argument. (C) Outside the Scope. This gives us a way to lower the risk of heart disease even more than taking vitamin C alone, but it still doesn’t change the fact that vitamin C reduces heart problems. The argument goes along unaffected. (E) 180. This is the opposite of what we want. This provides yet another reason why vitamin C is good for one’s health, thus strengthening the argument. 21. (D) Flaw Use information from the question stem to help you deduce the correct answer. According to the question, we’re looking for something that Boris fails to do in his response to George. George wants to know why people are now so interested in learning ballroom dancing when it wasn’t very popular in the 1980s and early 1990s. Boris responds by saying that people started learning certain ballroom dances in 1995 and the popularity of ballroom dancing caught on from there. So what did Boris fail to do? Well, he gave George a more specific starting date for the interest in ballroom dancing, but he still didn’t answer the big question: why the interest in dancing in the first place? (D) points out this unanswered query. For the record:

(C) Outside the Scope. There’s no need to connect ballroom dancing to any other form of dancing—prevalent or not. The fact is, ballroom dancing is of interest now, and other types of dancing are irrelevant to the question. (E) Extreme. Since George never asked about all types of ballroom dancing, just ballroom dancing in general, there was no need for Boris to account for all types. 22. (B) Inference EXCEPT Like in Logic Games, take a second to characterize Inference EXCEPT questions to avoid choosing a wrong answer. Four of the answers to this question will be possible (“could be true”). The one correct answer will be the one that is the exception—the one that absolutely cannot be true (i.e., must be false) or directly contradicts the stimulus. When looking for something that absolutely must be true or must be false, it helps to find some strong logical statements. In this stimulus, scientists are talking about two groups of hominids: Neanderthals and Cro-Magnons. We’re then told that both groups lived in different environments, but had tools that were exactly alike. Then comes the most important phrase of the stimulus: “only if they faced the same daily challenges… would they have used such similar tools.” Logically, this states that people who use the same types of tools must face the same daily challenges. Combining this with what we know, we have two groups of hominids who lived in different environments but used the same types of tools, which means that they must have faced the same daily challenges. The last sentence just tells us that the two groups have morphological differences, yet are probably still members of the same species. So, it’s off to the answers to find something that is in complete contrast to what we know. Remember to eliminate answer choices that could be supported by the stimulus. (A) This answer is actually supported by the last sentence. We have two groups with morphological differences, yet it’s suggested that they are the same species. (B) Wait a minute. We just learned about two groups of hominids who lived in different environments but, by the logic provided, must have faced the same daily challenges. Two environments—same daily challenges. Saying that daily challenges are unique to one environment is in complete contrast to this. Therefore, (B) is the exception, making it the correct answer. For the record:

39

LSAT Practice Test Explained

(C)The morphological differences between Neanderthals and Cro-Magnons are described in the statements as “minor.” However, without any description of the differences between Cro-Magnons and modern humans, we cannot say for sure whether those differences are greater or not. So, this could be true for all we know. (D) The fact that Cro-Magnons and Neanderthals both used the same tools is used as evidence here to suggest that they are the same species. So, a general rule that states that using similar tools is required to be of the same species is possible. It doesn’t have to be true, but it could be—and that’s enough to eliminate this answer. (E) We’re told that these two groups lived in different environments, so it’s not a stretch to say that they could have been geographically isolated from each other. 23. (E) Principle Be sure to take note of all key terms provided in the stimulus. In the stimulus, we are given two principles regarding summer weather. The first states that any day with intermittent winds and temperatures below 84 degrees all afternoon is called “pleasant.” The second states that any humid day with no wind or temperatures above 84 degrees all afternoon is called “oppressive.” We need to find the one answer that conforms to these definitions. (A) This day satisfies the “pleasant” condition of staying below 84 degrees all afternoon. However, the principle says we can call the day pleasant if there are also intermittent winds. This day had no wind, so we don’t satisfy the conditions of the principle. Eliminate. (B) With temperatures above 84 degrees all afternoon, this day certainly seems to meet the criteria for “oppressive.” Unfortunately, the “oppressive” principle only applies to days with high humidity. Since this day had low humidity, the “oppressive” principle cannot be applied. Eliminate. (C) This day has the sufficient intermittent winds. However, the temperature remained at a constant 84 degrees all afternoon. A day is considered pleasant if the temperature stays below 84 degrees. This doesn’t match the principle, and is therefore not the correct answer. Eliminate. (D) The conditions to make a day “oppressive” are either temperatures above 84 degrees or no wind. This day never had temperatures above 84 and had intermittent winds. There’s nothing to justify calling this day “oppressive” as per the principle. Eliminate. (E) This is a humid day with no wind. Regardless of the temperature, the lack of wind on a humid summer day qualifies as “oppressive” according to the second principle. That’s enough for a right answer.

40

24. (D) Parallel Reasoning In Parallel Reasoning, watch out for answers that have similar structure but don’t have the same level of qualification as the original argument. The argument in the stimulus is pretty straightforward. A radio station has never finished better than fifth place in the ratings, so it won’t win first place this year. So, we’re looking for another argument that uses past performance to indicate a definite continuation of that trend in the next performance. (A) Here, the past trend is that every swan the author has seen is white. To be parallel to the original argument, we’d want the author to claim that the next swan spotted will also be white. However, this argument makes the extreme conclusion that all swans are therefore white. That’s like saying the radio station in the original argument will never win the ratings. That’s not what the original argument says, so this is not exactly parallel. Eliminate. (B) The past trend here is the coin coming up heads. Like the original argument, the next toss is predicted to be heads. However, unlike the original argument, the prediction is qualified: It says the next toss will probably be heads. The original argument had no such qualification, making these arguments unparallel. Eliminate. (C), (E) Neither of these answers provide a past trend. Instead, they provide rules. (C) states that all lions are mammals. (E) states that only trained swimmers are lifeguards. There is no such rule in the original argument, so these arguments are not parallel. Eliminate both. (D) Here we have a past trend: Stock prices have always been lower on Mondays. The author then predicts, with certainty, that the stock will be lower the next Monday. That’s a perfect match to the structure of the stimulus, so (D) is the answer. 25. (E) Assumption In hard-to-predict questions, the Denial Test can quickly verify a right answer or reject a wrong answer. The chef in this argument describes the first step in preparing mussels in a given recipe. The point of that step, as the chef tells us, is to eject the sand from the mussel. However, the chef has decided to skip this step since seafood market mussels don’t have sand. Seems fairly logical. So, what’s the assumption? The chef is using seafood market mussels for the recipe. That has to be true in order for the chef’s logic to hold. Scan the answer choices for a match to our assumption: (A) This is unimportant. It doesn’t matter how the farm-raised mussels are cleaned out. All that matters is that they are cleaned out, so the first step of the recipe can be skipped. Eliminate. (B) The only reason for cornmeal (as described by the chef) is to remove sand. Since it has no effect on other contaminants that we know of, this is not an issue. Eliminate.

Section IV: Logical Reasoning

(C) This is tempting, but is it really necessary? What if cornmeal did affect the taste? That might give the chef even more reason to skip that first step. So even if this answer weren’t true, the argument could still work. Therefore, it’s not a required assumption. Eliminate. (D) It doesn’t matter when the recipe was written. Even if it was written years after the availability of farm-raised mussels, the cornmeal step is still in there, and the chef would still probably skip it. Eliminate. (E) This makes sense. If the chef feels the sand removal step can be skipped, the chef must be using sand-free mussels (i.e., the mussels from the seafood market). This matches our prediction. But, if you’re still not convinced, try denying this answer. What if the chef wasn’t using mussels from the seafood market? In that case, the mussels are not guaranteed to be sand free and there’d be no apparent reason for the chef to skip the sand removal step. The argument wouldn’t make any sense. We can’t deny this answer; therefore, it must be assumed and is the correct answer.

41

LSAT Practice Test Explained

Ex. modern settings. The blend of English with local idiom and Yoruba grammatical constructs, in which adjectives and verbs are often interchangeable, (50) re-creates the folktales in singular ways. And, perhaps most revealingly, in the majority of Tutuola’s works, the traditional accents and techniques of the teller of Ex. folktales are clearly discernible, for example in the adoption of an omniscient, summarizing voice at the (55) end of his narratives, a device that is generally recognized as being employed to conclude most folktales.

Section V: Reading Comprehension Passage 1—Literary Works of Amos Tutuola Sample Roadmap

With his first published works in the 1950s, Amos Tutuola became the first Nigerian writer to receive T-1st Nig. wide international recognition. Written in a mix of writer praised standard English, idiomatic Nigerian English, and inventive literal translation of his native language, Yoruba, approach Tutuola’s works were quick to be praised by many literary critics as fresh, inventive approaches to the form of the novel. Others, however, dismissed his works as simple retellings of local tales, full of Cr: dismissive; unwelcome liberties taken with the details of the well-known story lines. However, to estimate properly rehashing & embellish Tutuola’s rightful position in world literature, it is Au—T’s works essential to be clear about the genre in which he significant wrote; literary critics have assumed too facilely that —not a novel writer he wrote novels. No matter how flexible a definition of the novel Au—T is not a one uses, establishing a set of criteria that enable Tutuola’s works to be described as such applies to his true novelist works a body of assumptions the works are not —T is teller of designed to satisfy. Tutuola is not a novelist but a folktales teller of folktales. Many of his critics are right to suggest that Tutuola’s subjects are not strikingly Au: agrees w/cr's original, but it is important to bear in mind that —not orig & whereas realism and originality are expected of the that’s expected novel, the teller of folktales is expected to derive subjects and frameworks from the corpus of traditional lore. The most useful approach to Au Tutuola’s works, then, is one that regards him as working within the African oral tradition. Within this tradition, a folktale is common property, an expression of a people’s culture and social circumstances. The teller of folktales knows that the basic story is already known to most listeners Characteristics of and, equally, that the teller’s reputation depends on the inventiveness with which the tale is modified and folktale teller in embellished, for what the audience anticipates is not African oral trad. an accurate retelling of the story but effective improvisation and delivery. Thus, within the framework of the basic story, the teller is allowed considerable room to maneuver—in fact, the most brilliant tellers of folktales transform them into unique works. Tutuola’s adherence to this tradition is clear : How T uses charact' specific episodes, for example, are often repeated for emphasis, and he embellishes familiar tales with personal interpretations or by transferring them to




< >




Section V: Reading Comprehension

Discussion Paragraph 1 introduces us to the Topic of this passage, Amos Tutuola. Immediately, two contrasting opinions are provided about Tutuola’s works: those that praise his inventive approach to the novel and those that dismiss him for simply rehashing and taking liberties with well-known stories. The author clearly believes that Tutuola’s works are significant and finishes the paragraph by pointing out that true understanding of Tutuola’s contribution to literature requires clearly defining the genre of Tutuola’s works. By claiming that critics “too facilely” assume Tutuola writes novels, the author sets the stage for a discussion of the proper classification of Tutuola’s works—and it probably won’t be novels. This discussion of genre will serve as the Scope of the passage. In addition, the Purpose also seems pretty evident: illustrate how Tutuola’s works can be better understood by classifying them under the proper genre. Sure enough, the first sentence of Paragraph 2 finds the author claiming that Tutuola won’t satisfy the definition of “novelist,” no matter how loose the definition is. Instead, the author offers an alternative classification in a sentence that clearly lays out the Main Point of the passage: “Tutuola is not a novelist but a teller of folktales.” He’s not supposed to be original (like a novelist would be). Instead, Tutuola should be regarded as working in the African oral tradition of storytelling. Paragraph 3 defines the characteristics of a folktale teller in this tradition. The stories are already known and shared by the community. The teller is praised not for the story itself, but for the delivery and embellishments made to the tale for the sake of good storytelling. In fact, improvisation and modifications to the stories are expected and even praised. Paragraph 4 shows how Tutuola utilizes these characteristics in his own works, thus solidifying the author’s Main Point. The author provides multiple examples of Tutuola’s adherence to the oral storytelling tradition (e.g., resetting stories in modern times, blending linguistic styles, incorporating traditional folktale telling techniques) as further evidence that Tutuola’s works should not be evaluated as novels, but as the literary equivalent of the art of telling folktales. 1. (B) Global Beware of answers that shift the focus away from the main Topic of the passage. The author’s claim in lines 20–21 (Tutuola is a folktale teller, not a novelist) serves as an efficient, concise paraphrase of the Main Point. That sentiment is echoed in (B). For the record: (A) FUD. There’s nothing inaccurate about this answer. However, it merely regurgitates some minor facts from the first paragraph without touching on the overall theme of classifying Tutuola’s works.

(C) 180. Sounds tempting, but this answer makes the mistake of suggesting that Tutuola writes novels. That goes against everything the author said in the first sentence of Paragraph 2. (D)Distortion. This answer distorts the debate mentioned in the opening paragraphs. However, the author implies that both groups of critics regard Tutuola’s works as novels. Only the author suggests that Tutuola’s works should be seen as folktales. (E) FUD. This answer makes folktales the center of attention, and that’s not what this passage is concerned about. Amos Tutuola is not used in this passage as merely an example of the genre of “folktale,” as this answer suggests. Instead, the genre of folktale is used as an applicable way of classifying the works of Tutuola. 2. (D) Logic (Parallel Reasoning) Don’t let the shift in Topic throw you when asked to find an analogous situation. For this question, we’re looking to match Tutuola’s writing approach to another author. Tutuola’s approach was outlined by a few examples in the fourth paragraph: He embellishes familiar tales with personal interpretations or by transferring them to modern settings. The best strategy here is to compare the answers to the examples in the passage to find out which one matches exactly. (A)This is the opposite of what we’re looking for. Tutuloa applied folktale conventions to his modern works, not the other way around. (B)Tutuola didn’t try to recreate elements of any literary style. (C) Tutuola may have blended languages and linguistic constructs, but nothing in the passage suggests that he combined characters from different tales in his storytelling. (D) In lines 46–47, Tutuola is described as transferring traditional tales to modern settings. That sounds exactly like a modern Irish author transplanting traditional Irish tales into contemporary settings. This is parallel and is thus the correct answer. For the record: (E)This answer tries to throw test takers off by mentioning the omniscient narrator that was used by Tutuola according to lines 53–55. However, that narrating voice is described as one that summarizes the story and the end of the narrative. More importantly, though, this answer discusses original stories. Tutuola didn’t tell original stories; he told well-known stories in an inventive manner. This is not a parallel circumstance. 3. (A) Inference Keywords typically provide an accurate signal of the author’s feelings. The author provides plenty of opinion about Tutuola’s works in the second paragraph. In Line 27, the author describes

43

LSAT Practice Test Explained

classifying Tutuola’s works as folktales in the African oral tradition as the “most useful approach.” That feeling is described perfectly by (A). For the record:

insistence that Tutuola didn’t write novels, this seems like something the author would never have claimed. As it turns out, this is the exception and is therefore the correct answer.

(B) Outside the Scope. The author never discusses any renewed interest in the study of oral traditions.

To find the other four answers, a good place to start looking for attributes of Tutuola’s works is the fourth paragraph. That’s where we find (A), repetition of ideas for emphasis (lines 43–45); (B), relocation to modern settings (lines 46–47); and (D), the omniscient narrator (lines 54–55). To find reference to Tutuola’s international recognition, as mentioned in (C), you have to jump all the way back up to the first sentence of the passage.

(C)180. The “however” in line 11 and the “but” in line 23 indicate that the author is actually rather displeased by the literary critics’ assumptions and oversights. (D) Outside the Scope. This answer seems to suggest that some people feel that translations are ruining the integrity of Tutuola’s works. There’s no suggestion that translations pose any problems, so the author never expresses any attitude about this. (E) Outside the Scope. This passage is entirely devoted to Tutuola and does not discuss or suggest any trends in literature outside of Tutuola’s own works. 4. (D) Detail Don’t let misapplied terms from the passage divert your attention from what the question is asking about. This question asks about the criticism of Tutuola’s works, which is described in lines 9–11. Basically, the criticism is that Tutuola simply retells well-known stories and takes unwelcome liberties with the details. That’s exactly what (D) says. For the record: (A)180. The author is the one who claims that Tutuola’s works are based on African oral tradition, not the critics. (B) 180. It is mentioned that Tutuola mixes languages, but this is not mentioned as a source of complaint. In fact, it’s mentioned in the same sentence as the critics who praise Tutuola’s works. (C) Outside the Scope. The author suggests that critics classify Tutuola’s works as merely novels, not a blend of styles. The real problem the critics have is not that fusing styles is detrimental, but that Tutuola fails within the defined notion of what a novel should be. (E)180. Again, the author suggests that critics do actually see his works as novels. Furthermore, there’s nothing in the passage to imply that Tutuola himself characterized his stories as novels. 5. (E) Detail EXCEPT In EXCEPT questions, recognizing the flaw in the right answer can sometimes be more effective than searching for the four wrong answers. Since the author discusses Tutuola’s works throughout the passage, one might expect this question to take a little more time than others. However, a quick glance through the answers might turn up a glaring problem with (E), which discusses turning folktales into novels. Given the author’s

44

6. (A) Logic (Function) When a portion of a sentence is referenced in a Function question, be sure to reread the entire sentence to understand the full context. This question wants to know why the author mentions a “corpus of traditional lore.” To understand this, it’s important to start back from line 21. That’s when the author talks about the critics who complain that Tutuola’s stories aren’t original. After the “but” in line 23, the author is making the claim that original stories are characteristics of “novels,” not of the kind of folktales that Tutuola wrote. The “corpus of traditional lore” is the collection of nonoriginal material that characterizes folktales. This characterization is what the author uses to differentiate folktales from novels, and that leads us to (A). For the record: (B) Outside the Scope. The author isn’t concerned with the value of the two genres. If anything, the author is more concerned with the value of Tutuola’s works as folktales, not as novels. (C) 180. The critics aren’t against blending genres. In fact, according to the author, they don’t even see Tutuola’s works as a combination of genres—they only see the works as novels. (D) Distortion. The corpus of traditional lore isn’t so much a characteristic of folktales as it is a basis. And novels (whether or not one considers them largely dissimilar to folktales) don’t really have a direct counterpart. Because of the expectation of originality in novels, novels lack such a collection of tales to borrow from. (E) Distortion. The author doesn’t want critics to analyze two genres. Instead, he wants them to analyze Tutuola’s works within the confines of just one of the two. 7. (C) Global Some passages will come with a Global question at the end of the question set. Grab this point first before tackling the more detail-oriented questions. The Purpose of the passage comes across pretty clearly in the author’s use of the words “essential” in line 13 and “most

Section V: Reading Comprehension

useful approach” in line 28. The author is recommending a better way to classify Tutuola’s works in order to better understand them. Also, the examples of Tutuola’s works in the last paragraph help provide specific evidence to back up that recommendation. So, a good prediction of the author’s Purpose would be: to use examples of Tutuola’s works to show how those works should be better classified for more proper evaluation. And that’s what (C) basically says. For the record: (A)FUD. This passage predominantly confines Tutuola’s works to the realm of folktales and doesn’t really ascribe any further range. (B), (D) Distortion. These answers misapply the focus of the passage onto the genres of literature when the passage is really concerned about the works of Tutuola. (E) FUD. This is covered in the first paragraph, but this answer pretty much ignores everything that comes afterwards.

45

LSAT Practice Test Explained

Passage 2—Inclusive Fitness Theory of Kin Recognition

that cannibal larvae are especially likely to be infected by eating diseased species members. The fact avoid eating (55) that this bacterium is more deadly when it comes from a close relative with a similar immune system each other suggests that natural selection may favor cannibals b/c it can kill them Sample Roadmap that avoid such pathogens by not eating kin. For tiger Ways of Kin salamanders then, kin recognition can be explained Mechanisms for recognizing kin are found throughout the plant and animal kingdoms, regardless Recog. are (60) simply as a means by which an organism preserves –save own common its own life, not as a means to aid in relatives’ survival. life of an organism’s social or mental complexity. Improvements in the general understanding of these (5) mechanisms have turned some biologists’ attention to Bio: Why? exist the question of why kin recognition occurs at all. One Reason: IFT response to this question is offered by the which was developed in the 1960s. The theory is based on the realization that an organism (10) transmits its genetic attributes to succeeding generations not solely through its offspring, but more generally through all of its close relatives. Whereas goes beyond the traditional view of evolution held that natural selection favors the continued genetic representation Nat. Sel.— not just (15) of individuals within a species that produce the offspring; all greatest number of offspring, the inclusive fitness relatives theory posits that natural selection similarly favors organisms that help their relatives, because doing so also increases their own total genetic representation. explains (20) The theory has helped to explain previously honeybee mysterious phenomena, including the evolution of evol. social insect species like the honeybee, most of whose members do not produce offspring and exist only to nurture relatives. Inclusive fitness theory has also been applied IFT → (25) may explain usefully to new findings concerning cannibalism within animal species. Based on the theory, cannibals cannibalism should have evolved to avoid eating their own kin because of the obvious genetic costs of such a Ex: Spadefoot (30) practice. Spadefoot toad tadpoles provide an illustration. Biologists have found that all tadpoles of toad tadpoles that species begin life as omnivores, feeding mainly on organic debris in their soon-to-be-dry pool in the –ensure, relatives survival desert, but that occasionally one tadpole eats another (35) or eats a freshwater shrimp. This event can trigger They eat w/in changes in the tadpole’s physiology and dietary preference, causing the tadpole to become larger and species, but not own kin exclusively carnivorous, feasting on other animals including members of its own species. Yet the (40) cannibals have a procedure of discrimination whereby they nip at other tadpoles, eating nonsiblings but releasing siblings unharmed. This suggests that the inclusive fitness theory offers at least a partial answer to why kin recognition develops. Interestingly, a except when (45) cannibal tadpole is less likely to avoid eating kin starving when it becomes very hungry, apparently putting its own unique genetic makeup ahead of its siblings’. But there may be other reasons why organisms Other recognize kin. For example, it has recently been reasons (50) found that tiger salamander larvae, also either Ex: Tiger omnivorous or cannibalistic, are plagued in nature by salamander a deadly bacterium. Furthermore, it was determined larvae 46

Section V: Reading Comprehension

Discussion This passage starts off with a typical LSAT contrast. Scientific understanding of how an event occurs is improving, but there’s still a big question of why that event occurs. In this passage, the event is kin recognition (the Topic of the passage). And, like most Natural Sciences passages, we are introduced to one explanation. In this case, the offered explanation is the inclusive fitness theory (the Scope of the passage). The bulk of the first paragraph is some background information on the theory: Unlike traditional theories in which natural selection favors individuals with the most offspring, the inclusive fitness theory states that animals recognize family because natural selection favors organisms that nurture their relatives. The reason is that doing so aids the success of the total genetic makeup of an organism’s family. The paragraph ends by showing how this theory has already been applied to help explain honeybees that don’t reproduce but exist merely to nurture relatives. Paragraph 2 offers another application of the inclusive fitness theory—within species that experience cannibalism. The theory again explains why animals would want to recognize their own kin. Eating kin would be detrimental to the success of one’s genetic lineage. To illustrate this, the author provides an example of spadefoot toad tadpoles. In some cases, one tadpole will accidentally eat another (or something similar), which can eventually lead that tadpole to feast frequently on fellow tadpoles. Applying the inclusive fitness theory, these cannibalistic tadpoles would want to recognize kin because they would rather eat nonrelatives than potentially disrupt the genetic survival of their family’s characteristics. While the theory seems to apply pretty well in general, an interesting note pops up at the end of the paragraph suggesting that in extreme circumstances (when tadpoles get very hungry), tadpoles may eat siblings after all to save themselves— contrary to what inclusive theory would suggest. The “but” at the beginning of Paragraph 3 suggests more problems for the inclusive fitness theory: another reason for kin recognition other than genetic survival. Sure enough, the author provides the example of tiger salamander larvae. These larvae can also become cannibalistic. However, they are also plagued by a deadly bacterium. As it turns out, it’s more deadly if the larvae get it from eating family than if they get it from another source. So, unlike the tadpoles, the salamander larvae don’t avoid eating kin because of the desire to maintain genetic lineage, they avoid eating kin for personal reasons—they don’t want to die. There’s a lot of scientific jargon, but the key to success here is to break things down into simple language. The Purpose of the passage is to discuss the inclusive fitness theory and show examples of how it can be applied and how it sometimes can’t. With no strong author opinion, the Main Idea is just that the inclusive theory suggests that kin recognition developed

for genetic preservation, and while it can be applied in some instances, other instances suggest that other explanations for kin recognition may still exist. 8. (A) Global Structural keywords can often provide a pretty accurate basis for answering Main Point questions in passages without a strong author opinion. There’s no strong author opinion here, so the answer to the Main Point is going to be equally neutral. The passage begins with an introduction of a theory that “posits” (line 17) that kin recognition developed to aid an organism’s total genetic representation. The theory is then applied to honeybees (lines 20–24) and tadpoles (Paragraph 2). The “but” in line 48 really drives home the final point—the inclusive theory can’t explain everything (particularly not the salamander larvae). So, we want an answer that expresses the theory properly and shows how it can apply, but not always. (A) is a perfect fit. This answer accurately describes the inclusive fitness theory, mentions the supporting examples, and (most importantly) mentions the fact that it doesn’t necessarily explain everything. For the record: (B) Outside the Scope. This passage doesn’t actually discuss the mechanisms used to recognize kin. It only discusses the reasons for kin recognition. (C) 180. Actually, this is contrary to the facts. The salamander research would more likely support any potential hypothesis of traditional evolutionists (as described in lines 13–16). They feel that natural selection favors individual organisms— which might explain why salamanders are only looking out for themselves. The salamander research runs contrast to the inclusive fitness theory, which posits natural section favoring an organism’s total genetic representation (including family). (D) FUD. While this may be accurate, it focuses too much on the salamander research and the traditional theory of natural selection. It completely ignores the inclusive fitness theory, which is the primary focus of the entire passage. (E) Extreme. The third paragraph alone shows that the inclusive fitness theory does not (as this answer states) fully explain kin recognition. 9. (C) Detail In heavily detailed passages, try not to answer questions based on memory because wrong answers will usually include misapplied details. Spadefoot tadpoles are described throughout the second paragraph. There’s a lot of information, so predicting the answer may not be feasible. Instead, let’s go through the answers and research only the likely candidates.

47

LSAT Practice Test Explained

(A) Doesn’t seem familiar. Lines 39–42 suggest that they can differentiate between siblings and nonsiblings, but there’s nothing stated about recognizing other carnivores. Eliminate. (B) Lines 37–39 describe how tadpoles can grow to become exclusively carnivorous, but there’s nothing about the tadpoles being selective about eating carnivores versus omnivores. Eliminate. (C) According to lines 35–38, the eating of another tadpole can change a tadpole’s dietary habits (turning it carnivorous), which causes the tadpole to become larger. That detail about a change in body size makes this answer a match. For the record: (D) This is the complete opposite. The behavior of the tadpoles is mentioned as direct support for the inclusive fitness theory. (E) A slight distortion. It’s not the carnivorousness that develops to protect the evolutionary success of the species. It’s the ability to recognize kin that develops to protect the species. 10. (D) Inference In detail-oriented passages, answers to questions about the author’s opinion will most likely be found around major keywords. According to the author, the inclusive fitness theory “has helped to explain” (line 20) and can be “applied usefully to” (lines 25–26) some instances of kin recognition. “But” (line 48), there may be other explanations for kin recognition. That’s (D). For the record: (A)Quite the opposite. According to the first paragraph, understanding of mechanisms has increased despite a lack of explanation for why kin recognition occurs.

(A) This seems like a good candidate. Just as we were being lulled into believing that the inclusive fitness theory was a winner, along comes this fact that throws a monkey wrench into the system. This is a match to our prediction. For the record: (B) Outside the Scope. The fact in the last sentence is referring only to the spadefoot toad tadpoles, and makes no comparison to other species. (C) 180. This “interesting” note runs contrary to everything that came before it. (D) Extreme. While it may question the validity of the inclusive fitness theory, it doesn’t necessarily make the tadpoles’ behavior unexplainable. In fact, it is explained—they’re very hungry and are more interested in saving themselves. (E)180. The information that precedes the last sentence is not less relevant. It’s plenty relevant as support for why the inclusive fitness theory is plausible. 12. (A) Inference Beware of answer choices that discuss information Outside the Scope of the passage.

(B)Again, opposite of what the author states. Lines 12–19 directly show how the inclusive fitness theory runs in contrast to the traditional evolutionary theory.

The mechanism of a tadpole for recognizing kin is described in lines 39–42. It involves nipping at other tadpoles to determine whether they are siblings or not.

(C) There is no suggestion on the author’s part that theories need to ignore any characteristics of any organisms.

(A) is definitely supported by the information in the passage. Nipping on other tadpoles is not a visual act, so the mechanism seems to be at least partially based on taste or touch. For the record:

(E) The author doesn’t mention any current theories outside of the inclusive fitness theory. However, by virtue of the last paragraph conflicting with the inclusive fitness theory, it’s evident that the author would not find that theory to be the most successful. 11. (A) Logic (Function) When a sentence is presented in contrast to previous sentences, expect a wrong answer to be completely opposite of what you’re looking for. To understand the function of the last sentence of the second paragraph, you need to remember the function of everything that came before it. The bulk of the second paragraph was about the tadpoles that avoided eating siblings in order to

48

improve the survival of their genetic lineage. This example was provided in support of the inclusive fitness theory. However, the last sentence begins with “interestingly,” suggesting that not all is as it seems. Sure enough, just as we’re being led to believe that tadpoles would spare family members to protect the genes, we learn that some tadpoles will eat their siblings after all—so as not to die from hunger. So, the Purpose seems to provide an instance that goes contrary to the other evidence, making the inclusive fitness theory seem inapplicable in certain cases. Scan the answer choices for a match:

(B) Outside the Scope. We know this is the behavior of cannibalistic tadpoles, but we don’t know what noncannibalistic tadpoles do. For all we know, they may nip at other tadpoles, too. In fact, according to the passage, tadpoles can become cannibalistic after first accidentally eating another tadpole. So, it doesn’t seem much of a stretch to consider that some noncarnivorous tadpoles might also nip other tadpoles. (C) Outside the Scope. Based on the last sentence, we know that tadpoles will still eat siblings in extreme circumstance. However, that’s not because the mechanism doesn’t work— it’s because of hunger. There’s no support for this.

Section V: Reading Comprehension

(D)180. The tadpoles utilize this mechanism after changing physiologically and becoming carnivorous.

necessarily a catalyst for dismissing traditional theory. Eliminate.

(E) 180. Line 32 states that all of these tadpoles start life as omnivores. Furthermore, we don’t have any information about other species beyond the tadpoles, so we can’t infer this.

(D)is a grand distortion. The inclusive fitness theory (which helps explain the honeybees’ behavior) still states that evolution proceeds by natural selection—just in a different way from what traditional theories state. Eliminate.

13. (B) Detail

(E)is a match. It was mysterious under traditional theories, so some supplement (in this case, the inclusive fitness theory) was needed to dispel some of the mystery.

Answers that incorporate figures and dates often misapply that information. Be wary of these answers. Most of the passage involves theories to explain kin recognition. Mechanisms are mostly discussed early in the first paragraph. In fact, the answer to this question is clearly presented in lines 4–7: Improvements in understanding these mechanisms have led to questions about why they occur. That’s (B). For the record:

15. (C) Logic (Weaken the Argument) Classic ways of weakening arguments in Logical Reasoning will work equally effectively in Reading Comprehension Logic Reasoning questions.

(C) FUD. Some people might be tempted by this because it uses “1960s” from the passage. However, the 1960s is only when the inclusive fitness theory was developed. For all we know, there were many other theories developed long before the 1960s.

In the last sentence, the author concludes that kin recognition in tiger salamanders can be explained as a means for preserving their own life and not as a means for aiding their relatives’ survival. While the evidence regarding the deadly bacterium definitely supports kin recognition being used to preserve the individual, there is no evidence to say that kin recognition is not used to aid relatives’ survival. The author ignores the possibility that kin recognition may serve to protect oneself and one’s relatives. To weaken the claim, the correct answer choice will show a way that a tiger salamander would use kin recognition to protect someone other than itself.

(D) Outside the Scope. While the passage may suggest that we still don’t have a full explanation of their purpose, the author never states that this is because we need more understanding of the mechanisms.

(A) This has no relevance to the argument. What’s important is not whether the disease affects cannibalistic or noncannibalistic salamanders, but whether or not it affects kin and kin recognition. Eliminate.

(E) Outside the Scope. While the passage may discuss different purposes for the mechanisms, there’s no description of how similar or different the mechanisms operate.

(B) This discusses what makes salamanders carnivorous or omnivorous, but it has no bearing on why salamanders recognize kin. Eliminate.

(A) Outside the Scope. The first sentence says that mechanisms exist throughout the animal kingdom, regardless of complexity. However, that doesn’t mean that we should assume they have a similar purpose in all species. In fact, by the two examples in the second and third paragraph, we get the sense that mechanisms can serve multiple purposes.

14. (E) Inference Taking note of where all examples appear, even seemingly minor ones, can save research time when questions ask about them. The honeybees were mentioned at the end of the first paragraph. According to the paragraph, the honeybees’ behavior of nurturing relatives was “previously mysterious,” but the inclusive fitness theory helped explain it. We want an answer choice that’s consistent with this prediction. (A) The behavior was known, it was just unexplained. Eliminate. (B)There’s no discussion of this kind of reciprocal nurturing. Eliminate. (C) While the new theory helps to explain this behavior, the behavior was simply described as previously mysterious. This suggests the behavior was merely an enigma and not

(C) gives us a fact that would weaken the author’s claim. If this were true, then kin recognition would provide a way for salamanders to protect their offspring—thus making kin recognition valuable beyond survival of the individual. By protecting potential family, this weakens the author’s claim that kin recognition is simply a self-serving device. For the record: (D) This answer misapplies some information from the first paragraph about the number of offspring (part of traditional evolutionary views). However, once again, the number of offspring a salamander has is not directly relevant to the reason for being able to recognize kin. (E) Even if this were true, according to the passage, they’re still immune to the one deadly bacterium that’s mentioned. And, because that particular bacterium is more deadly when consumed through kin, the greater immunity to other diseases isn’t necessarily relevant to kin recognition. It’s good to note for this question that the correct answer is the only one that focuses on kin recognition, the Topic of the argument the question asks us to weaken.

49

LSAT Practice Test Explained

Passage 3—Definition of National Minority (Comparative Reading) Sample Roadmap

(5)

(10)

(15)

(20)

(25)

(30)

(35)

(40)

(45)

50

Passage A It is most commonly applied to 1 groups of persons—not necessarily citizens—under the jurisdiction of one country who have ethnic ties to another “homeland” country, or 2 groups of citizens of a country who have lasting ties to that country and have no such ties to any other country, but are distinguished from the majority of the population by ethnicity, religion, or language. The terms “people” and “nation” are also vaguely defined in international agreements. Documents that refer to a “nation” generally link the term to the concept of “nationalism,” which is often associated with ties to land. It also connotes sovereignty, for which reason, perhaps, “people” is often used instead of “nation” for groups subject to a colonial power. While the lack of definition of the terms “minority,” “people,” and “nation” presents difficulties to numerous minority groups, this lack is particularly problematic for the Roma (Gypsies). The Roma are not a colonized people, they do not have a homeland, and many do not bear ties to any currently existing country. Some Roma are not even citizens of any country, in part because of their nomadic way of life, which developed in response to centuries of fleeing persecution. Instead, they have ethnic and linguistic ties to other groups of Roma that reside in other countries. Passage B Capotorti’s definition of a minority includes four empirical criteria—�a group’s being numerically smaller than the rest of the population of the state; �their being nondominant; �their having distinctive ethnic, linguistic, or religious characteristics; and �their desiring to preserve their own culture—and one legal criterion, that they be citizens of the state in question. given the previous nomadic character of the Roma, that they still cross borders between European states to avoid persecution, and that some states have denied them citizenship, and thus minority status. Because this element essentially grants the state the arbitrary right to decide if the Roma constitute a minority without reference to empirical characteristics, it seems patently unfair that it should be included in the definition. However, the Roma easily fulfill the four objective elements of Capotorti’s definition and should, therefore, Numerically, they are nowhere near a majority, though they number in the hundreds of thousands, even millions, in some states. Their

(50) nondominant position is evident—they are not even -numerically acknowledged as a minority in some states. The Roma not a maj. -distinct have a number of distinctive linguistic, ethnic, and character. religious characteristics. For example, most speak Romani, an Indo-European language descended from Ex (55) Sanskrit. Roma groups also have their own distinctive lang. legal and court systems, which are group oriented rather than individual-rights oriented. That they have preserv. shows preserved their language, customs, and identity desire to through centuries of persecution is evidence enough preserve culture (60) of their desire to preserve their culture. No univ. def of NM 2 applications of NM terms are vague Problem e.g. Roma

Cap’s def. of NM: 4 emp 1 leg

Problem e.g. Roma meet only 4/5 crit.

Au—Roma meet def. of NM

Section V: Reading Comprehension

Discussion The Topic of Passage A is presented immediately in the first sentence: national minority. According to the author of Passage A, there is no universally accepted definition for the term “national minority.” This lack of definition constitutes the Scope of Passage A. The author offers two frequent applications of the term, but further complains that other terms (e.g., “nation” and “people”) used in defining national minority are similarly vague. In the second paragraph, the author discusses how this lack of accepted definitions is potentially harmful, particularly to one group: the Roma. The author’s Purpose is merely to describe this problem and how it affects the Roma in particular. The Main Point of Passage A is that the lack of an agreed upon definition of “national minority” (as well as “nation” and “people”) has negative consequences for certain groups, including the Roma. As would be expected, the Topic of Passage B is also national minority. However, the author of Passage B isn’t concerned about the lack of a definition. The Scope of Passage B is one specific definition of national minority: Capotorti’s definition. Passage B starts off by laying out the five criteria for Capotorti’s definition: four empirical ones and one legal. The author of Passage B then discusses a problem with the legal criterion, particularly in respect to the Roma. Similar to Passage A, one purpose of Passage B is to describe how a national minority definition is problematic for the Roma. However, the second paragraph provides a more refined Purpose: show how the Roma should qualify as a national minority using the remaining, more objective criteria. The Main Point of Passage B is that the legal criterion of Capotorti’s definition poses a problem to the Roma by denying them status as a national minority despite their fulfillment of the remaining objective criteria. Before going to the questions, it’s important to note the relationship between the two passages. Both are concerned with the definition of national minority—although one is concerned about the lack of an accepted definition and one is concerned about the criteria of one definition in particular. More notably, both passages are concerned with how these issues cause problems for the Roma. 16. (E) Global When a Global question in Comparative Reading applies to only Passage A, it may be wise to answer that question before even reading Passage B. The Main Point of Passage A was laid out in a fairly straightforward manner: The lack of definition for “national minority” (as well as “nation” and “people”) has led to problems for certain groups, particularly the Roma. That’s exactly what (E) says. For the record: (A) Outside the Scope. The author doesn’t mention a conflict of definitions. The problem is caused by the lack of one universally accepted definition.

(B) FUD. While this is true, this answer ignores the entire reason why they’re not considered a minority. It leaves out the discussion of vague definitions that constituted the entire first paragraph. (C) 180. This reverses the causality. It’s the lack of an accepted definition that caused problems in applying terms to the Roma, not the other way around. (D) Extreme. The author complains that there’s no universally accepted definition, but never goes so far as to say no definition will ever apply to the Roma. 17. (C) Inference When asked to define a common word in Reading Comprehension, context is vital to avoiding wrong answers that provide common definitions. In both passages, the word “problematic” is used to describe how the Roma are affected by the definition (or lack of definition) of national minority. In Passage A, the lack of definition causes problems for the Roma and in Passage B, the legal criterion of the definition causes problems for the Roma. In both cases, “problematic” refers to causing problems, which is synonymous with (C). For the record: (A) Outside the Scope. Neither passage discusses a debate over the concept of national minority. (B) Extreme. The lack of definition may be considered confusing and unclear to the author of Passage A, but the definition is crystal clear in Passage B—it just causes problems. (D)Outside the Scope. Solving the problem isn’t a primary concern in either passage. Additionally, given that the author of Passage B feels that the Roma satisfy all the criteria except for one, the problem doesn’t seem all that difficult to solve. In fact, at the end of the first paragraph of Passage B, the author even implies the easy solution: exclude the legal criterion. (D) Extreme. Again, there’s nothing really incoherent about the definition provided in Passage B. It just causes problems in its inapplicability to the Roma. 18. (D) Detail EXCEPT In Comparative Reading passages, when a Detail question refers to one passage, expect an answer that appears in the other passage. Four of the answers to this question will be found in Passage A and one will not. There’s a good chance that the right answer will appear in Passage B. If you tackled this question before reading Passage B, the right answer would stand out as something you never read. However, if you read both passages, chances are you want to make sure the four wrong answers all appeared in Passage A. (A)This pretty much rewords the last sentence of Passage A. Eliminate.

51

LSAT Practice Test Explained

(B)The nomadic way of life is directly mentioned in line 24 of Passage A. Eliminate.

definition of that term under a particular set of criteria (here, the “union’s criteria”). For the record:

(C) Line 21 tells us they have no homeland. Eliminate.

(B) Distortion. The problem with this answer is in the first article. The author of passage A doesn’t feel that there is an accepted definition of national minority. That doesn’t match the first article’s suggest that there are accepted “current criteria.”

(D)Population numbers aren’t mentioned in Passage A. In fact, this detail comes from lines 48–49 of Passage B. This is the exception and is therefore the correct answer. For the record: (E)By saying the lack of definition is “particularly problematic for the Roma” in lines 19–20, this matches the claim that the Roma are worse off than other groups. 19. (E) Inference Recognizing the relationship between the two passages can lead to quick points. In both passages, the Roma are presented as a specific group that experiences problems due to issues with the international definitions of national minority. We want an answer choice that conveys this shared attitude. (A) It’s the author of Passage B who explicitly complains about the states’ “arbitrary right to decide if the Roma constitute a minority” (lines 40–41). Eliminate. (B)Both authors use the Roma as a noteworthy example of the problems of international definitions of national minority. Eliminate. (C) If anything, the author of Passage A seems more inclined to argue that the Roma are an example of a group that doesn’t constitute a nation. This is potentially a shared consideration and is not supported by the content of the passage. Eliminate. (D) Both passages are concerned about international definitions. Neither passage is concerned about political issues. Eliminate. (E) This is a perfect match that accurately expresses both authors’ shared idea that the Roma illustrate relatively distinctive problems with national minority definitions. 20. (A) Logic (Parallel Reasoning) When considering the relationship between both passages, stay focused on the subtle differences between the Scope and the Purpose. In the original passages, both passages are concerned with how the Roma are affected by national minority definitions. However, while Passage A was primarily concerned with how the Roma were affected by a vague definition, Passage B was concerned with one definition in specific and had the additional purpose of showing how the Roma really should satisfy that definition. The right answer to this question should apply the same conceptual purposes to an entirely different situation. (A) This matches up nicely. Here, instead of a group of nomadic people, we have welders. Like Passage A, the first article is concerned with a vague definition of a term (here, “technical”) that affects the group. And, like Passage B, the second article argues that the group should satisfy the

52

(C) Distortion. The author of Passage A doesn’t want to “revise” any definition, as the first article in this answer wants to do. The author of Passage A wants to come up with a definition that everyone can agree on. (D) Outside the Scope. The first article here makes an attack on the people enforcing a set of laws (citing “disregard”), which the author of Passage A never does. (E) Outside the Scope. These two articles express two differing opinions on the relationship between two terms: professionalism and technical expertise. The original two passages don’t have such a level of disagreement and are both only concerned primarily with one term: national minority. 21. (B) Logic (Principle) When asked to find an answer that applies to both passages, watch out for answers that seem to clearly apply primarily to just one. Both passages are concerned with how the Roma are affected by issues concerning international definitions of national minority. In both cases, the Roma are negatively affected because they aren’t technically covered by current definitions. The correct answer choice will get across the idea that international issues can have a negative effect on groups that aren’t definitely covered by the rules. (A) This answer works great for Passage A, but the author of Passage B doesn’t have any concern about vague definitions. Eliminate. (B) This fits both situations very well. Specifically, in both passages, the Roma aren’t officially recognized as a national minority and that has a detrimental effect on their interests. For the record: (C) Neither author would seem to have problems with provisions that apply to minority groups. They just have problems with specific provisions regarding the definition of national minority. In fact, given both authors’ concern for the Roma, they would both probably favor provisions that apply to minority groups. (D) The author of Passage B mentions that the Roma have legal and court systems in their jurisdictions, so this would support that author’s reasoning. However, the author of Passage A has no such concern. Therefore, this principle doesn’t apply to both authors. (E) Neither author seems to mind that the Roma aren’t considered citizens. The problem is that they’re denied status as a national minority.

Section V: Reading Comprehension

Passage 4—Women’s Education in France Sample Roadmap

(5)

(10)

(15)

(20)

(25)

(30)

(35)

(40)

(45)

(50)

compulsory attendance for all students. In order to pass these reforms, the government needed to (55) demonstrate that its new standards were rooted in a long philosophical, political, and pedagogical tradition. Various of the resulting institutions also made claim to revolutionary origin, as doing so allowed them to appropriate the legitimacy conferred (60) by tradition and historical continuity. —obst. overcome

During most of the nineteenth century, many French women continued to be educated according to models long established by custom and religious tradition. One recent observer has termed the failure to institute real and lasting educational reform at the Trad. ed. of Fr. women in 19th C. end of the eighteenth century a “missed opportunity”— for in spite of the egalitarian and secular aims of the French Revolution in 1789, a truly nondiscriminatory education system for both women and men would not be established in the country until the 1880s. However, legislators had put Critic: needed reform forth many proposals for educational reform in the years just after the revolution; two in particular attempted to institute educational systems for women Au: 2 prop. made that were, to a great extent, egalitarian. The first of these proposals endeavored to replace the predominantly religious education that women originally received in convents and at home with reformed curricula. More importantly, the proposal First prop: insisted that, because education was a common good —less rel emph. —available to both sexes that should be offered to both sexes, By the same token, teachers would be drawn from both sexes. Thus the proposal held it essential that schools for both men and women be established promptly throughout the country and that these schools be public, a tangible sign of the state’s interest in all of its citizens. One limitation of this proposal, however, was that girls, —Problem unlike boys, were to leave school at age eight in order to be educated at home in the skills necessary for domestic life and for the raising of families. The second proposal took a more comprehensive approach.  It advocated equal education for women Second prop: and men on the grounds that women and men enjoy —coed the same rights, and it was the only proposal of the —radical time that called for coeducational schools, which were presented as a bulwark against the traditional gender roles enforced by religious tradition. In other respects, however, this proposal also continued to —Problem define women in terms of their roles in the domestic sphere and as mothers. That neither proposal was able to envision a system of education that was fully equal for women, Prop. failed b/c cult. and that neither was adopted into law even as such, & pol. obst. bespeaks the immensity of the cultural and political obstacles to egalitarian education for women at the time. Nevertheless, the vision of egalitarian educational reform was not entirely lost. Nearly a century later, in the early 1880s, French legislators late 19th C—reform recalled the earlier proposals in their justification of began to take new laws that founded public secondary schools for shape women, abolished fees for education, and established

53

LSAT Practice Test Explained

Discussion The opening paragraph describes how the education of women in 19th-century France (the Topic) followed longstanding traditions. That’s followed by a complaint by one observer that France missed an opportunity during that time after the Revolution to make education less discriminatory. Once the author uses the word “however” in line 11, it’s clear the author has a bone to pick with that particular observation regarding educational reform (the Scope). In fact, the author points out that the legislator did, in fact, try to reform education during that time, and the author makes mention of two proposals in particular. These proposals are outlined in Paragraph 2. The first proposal suggested that women’s curricula be revised to have less religious emphasis, instruction be available to people (and taught by people) of both sexes, and public schools be established for both men and women. Another “however” in line 28 indicates the problem with this first proposal: Girls would be removed from this education at the young age of eight so that they could learn domestic skills at home. The second proposal was a little more radical and recommended coed school where women and men had the same rights to education. Unfortunately, another “however” in line 39 details the problem with the second proposal: Women were still defined as having domestic roles in society. After comparing the two proposals (the first part of the passage’s Purpose), Paragraph 3 suggests that their failure showed the immensity of the obstacles that needed to be overcome. However, those obstacles were eventually overcome (as indicated by the “nevertheless” in line 47). The rest of the paragraph explains how the two proposals were recalled and used as a foundation for later reforms because they were then considered legitimate examples of historical tradition. So the overall Purpose was not only to compare the two proposals, but also to show how they eventually helped serve as the foundation for later reform. The Main Point is that legislators didn’t necessarily miss an opportunity for reforming women’s education after the Revolution. They had proposals to reform education, but weren’t successful due to obstacles they couldn’t overcome until nearly a century later. 22. (C) Inference Use context clues in the question stem to make research more efficient. This question is looking for information about legislators who passed education laws in the 1880s. That wording (especially the use of the date) leads us right to lines 48–53. In those lines, we learn that not only did they establish schools for women, but they also abolished education fees and made attendance mandatory. Armed with that information, it’s time to go through the answers.

54

(A) While one may assume that these legislators started to shy away from the idea that women were solely defined as domestic, that doesn’t suggest a complete removal of domestic skills from education. Eliminate. (B) In the lines that end the passage, it seems that, on the contrary, legislators were well aware of past obstacles and had to work hard to make sure that reforms were accepted. Eliminate. (C) By eliminating education fees, it certainly seems that the legislators had economic motives in addition to genderrelated motives in their reforms. This is the correct response. For the record: (D)Since these legislators also had to conform to standards of political tradition (as suggested in lines 54–57), there’s no support to the idea that they were more willing to compromise such traditions than earlier legislators. (E) This answer seems to refer directly to the reduced religious emphasis as defined in the first proposal. However, since the legislators in the 1880s used that first proposal as part of the foundation of their reform, we can’t infer that they were more inclined to allow a religious role in education. 23. (E) Global A solid roadmap pays off big dividends with Global questions about organization. Recalling the structure of the passage, the author started by discussing the status of women’s education in the 19th century following the Revolution. After presenting a complaint about this status, the author provides two proposals that were presented to change that status. After discussing how these proposals indicated major obstacles, the author concludes by showing the eventual reform and how it came about. (E) captures just about everything, and all in the correct order. For the record: (A) Distortion. Everything seems to be pretty accurate here, but this answer fails on two accounts. First, the problems with the proposals are more pragmatic issues than they are inconsistencies. Second, this answer entirely ignores the last paragraph and the discussion of the eventual reform. (B) Outside the Scope. This answer falls apart almost immediately because gender equality is not really a focus of the discussion in the first paragraph. Furthermore, like (A), this answer fails to address the reform mentioned in the last paragraph. (C) FUD. Close, but the eventual change still required a break with tradition. This answer focuses too much on breaking with tradition rather than reforming the educational system. (D) Outside the Scope. Like (B), this answer fails by refocusing the first paragraph on discussing the egalitarian aims in France. Furthermore, any aims mentioned are not modified at the end.

Section V: Reading Comprehension

24. (A) Logic (Parallel Reasoning)

26. (E) Inference

Don’t let long question stems distract you from your goal.

Combine information in the passage to make powerful predictions on Inference questions.

Despite the length of the question stem, we’re merely looking for two proposals on housing reform that mimic the same logic as the two proposals on education reform presented in the passage. The first proposal suggested making education available to everyone—men and women. The second proposal suggested making education equal for everyone. We want a similar set of proposals that mimic the “something for everyone” and “everyone gets the same” concepts. (A) is an instant winner. For the record: (B) Distortion. The first proposal is a perfect match, but the second proposal focuses on improving the system rather than making it equal for everyone. (C) Distortion. The second proposal is a good match, but the first proposal offers a “those who can pay” qualification that isn’t found in the first education reform proposal. (D) Distortion. Again, the second proposal is a good match, but the first proposal focuses on improving the system rather than making it available to everyone.

The proposals are first mentioned at the end of Paragraph 1. Right before that, in lines 4–11, is the complaint of one observer about educational reform. The observer rued the lack of reform, “in spite of the egalitarian” aims of the French Revolution. The author’s “however” in line 11 suggests that the egalitarian aims were not a moot point. And the last sentence even reinforces the idea that these proposals were indeed egalitarian, as the observer would have expected. That leads to (E), which captures the egalitarian nature of the proposals. For the record: (A), (B) Outside the Scope. “Excesses” of the new government and the “power of education” are concepts that are not discussed in the passage. (C) 180. Seeing that both proposals failed, it’s hard to infer that they had much popular support. (D) Outside the Scope. This answer brings up other reforms not discussed in the passage.

(E) Outside the Scope. The first proposal focuses on cost, which is something the first educational proposal never does.

27. (C) Inference

25. (B) Detail

Not all questions will be predictable. Going through the answers will be your best bet on such questions.

Spotting keywords ahead of time can make significant points stand out. The second proposal is described in the second half of Paragraph 2. The word “only” in line 35 seems to be a good indication of a distinctive feature of that proposal. According to that line, the second proposal was the only proposal that called for coed schools. That’s a perfect match for (B). For the record: (A) 180. This is actually a feature of the first proposal (lines 21–23). (C) Outside the Scope. While that would be in contrast to the first proposal that removes girls from school at the age of eight, the author never states anything in the passage about “lifelong learning.” (D) Extreme. While the coed schools recommended in the proposal were described as going against religious tradition, the author never goes so far as to say the proposal looked to abolish religious schools. (E) 180. Again, it was the first proposal that called for public schools and then a return home to learn secular domestic skills.

The author doesn’t express much opinion in this passage. Most comments about the proposals were saved until the beginning of the last paragraph, in which the author expresses the failure of both proposals and how their failure illustrates the tremendous obstacles faced by reformers. We need to look out for wrong answers that distort this point of view. (A) Given that the second proposal is described as “a bulwark against the traditional gender roles enforced by religious tradition” (lines 37–38), it’s difficult to infer that the proposal was “modest.” Eliminate. (B) The author never suggests anything unethical about either proposal. Eliminate. (C) Both proposals are described as facing major obstacles, which speaks to them meaning well. And, by calling the problems “limitations” in the second paragraph, the author is definitely suggesting that they failed due to the constraints at that time. This answer works fine. For the record: (D) The author presents an explanation of why they failed in the last paragraph, indicating that it’s not necessarily difficult to understand the reasoning. (E) According to the last paragraph, the proposals failed because of “cultural and political obstacles,” not because they weren’t comprehensive.

55

1-800-KAP-TEST | kaplanLSAT.com *LSAT is a registered trademark of the Law School Admission Council.